You are on page 1of 40

CMA 1 D

Domestic Insitutional [5] Source: CMA 0687 1-13


Antitrust laws
Environment of Business
A. Set maximum allowable prices for firms in
128 Questions oligopolistic industries.

B. Prohibit firms from collecting price information.


[1] Source: CMA 0685 1-31
Sunset laws refer to C. Require firms with high earnings to provide price
and quantity data to the Federal Trade Commission.
A. Government support for major business firms in a
state of decline. D. Prohibit firms from allocating customers.

B. Protection of employees nearing retirement.


[6] Source: CMA 0687 1-15
C. Regulatory statutes applicable to certain defense Tie-in sales (e.g., the sale of camera and film together) are
contractors. legal

D. A periodic review of and a fixed termination date A. If the tie-in is necessary to assure product quality.
for government programs.
B. If the tying product is patented.

[2] Source: CMA 1286 1-23 C. If used to facilitate price discrimination.


Currently, corporations are chartered by
D. Under all circumstances.
A. The federal government.

B. State governments. [7] Source: CMA 0687 1-22


The Federal Communications Commission
C. The Securities and Exchange Commission.
A. Allows free access to the radio spectrum.
D. The board of directors of the respective
corporations. B. Limits the number of television stations the major
television networks may own.

[3] Source: CMA 1286 1-28 C. Regulates the profits of radio and television
Which one of the following statements is a correct stations.
characterization of current federal policy toward political
action committees (PACs)? D. Censors program content on television.

A. Corporations may use company funds to organize


and administer a PAC. [8] Source: CMA 1287 1-14
An example of social regulation of business is
B. Corporations may make corporate contributions
to PACs. A. Tariffs on imported lumber products.

C. Unions may help organize PACs, but corporations B. Rules against discrimination in the hiring of
may not. employees.

D. Individuals may only make political contributions C. Enforcement of patent laws.


through PACs.
D. Antitrust rules agreements to allocate customers.

[4] Source: CMA 1286 1-26


The recent legal concept of strict liability in tort [9] Source: CMA 1287 1-16
Requirements by licensors and franchisors that their
A. Requires consumers to prove that producers were licensees and franchisees buy inputs from a particular
negligent. supplier

B. Requires the proof of breach of warranty. A. Are legal if they are necessary to assure product
quality.
C. Permits contributory negligence on the part of
consumers to be an acceptable defense by B. Are always illegal unless the input is patented.
manufacturers.
C. Are legal if used to facilitate price discrimination.
D. Tends to support plaintiff claims against
manufacturers of products that are judged to be D. Are legal under all circumstances.
inherently dangerous.
If Federal Aviation Administration regulation of airline
[10] Source: CMA 1287 1-17 passenger service increases the variable costs of individual
The intent of antitrust laws is to firms in the industry by requiring greater expenditures on
quality checks and maintenance, then
A. Establish a range of allowable profit rates for firms
in oligopolistic industries. A. In the long run, the industry will tend to have more
firms producing lower levels of output.
B. Prohibit firms from engaging in joint ventures with
foreign firms. B. Fixed costs will rise.

C. Require firms with high earnings to relinquish any C. Prices will fall in the long run.
exclusive patent rights which they own.
D. Prices will rise in the short and long run.
D. Prohibit agreements that limit individual firm
output.
[15] Source: CMA 1289 1-26
The two major functions of the Federal Trade Commission
[11] Source: CMA 1287 1-18 are
Which one of the following statements regarding the
application of antitrust rules to professional fees is true? A. Antitrust actions and consumer protection.

A. Most professionals, including physicians, lawyers, B. Antitrust actions and regulation of import quotas.
engineers, and accountants, are exempt from the
antitrust laws. C. Regulation of railroads and airlines.

B. Agreements to increase fees are legal if the quality D. Consumer protection and monitoring labor union
of services provided improves. practices.

C. Fee discrimination in any form is illegal. [16] Source: CMA 1289 1-24
All of the following are characteristic of the Taft-Hartley
D. Agreements to set either minimum or maximum Act except
fees are illegal.
A. The prohibition of a closed shop.

[12] Source: CMA 1287 1-19 B. The guarantee of the workers' right to organize.
The antitrust prohibitions against mergers that would tend
to lessen competition C. States are encouraged to pass right-to-work laws.

A. Are enforced solely by the Commerce D. The provision of descriptions of unfair labor
Department. practices committed by unions.

B. Are enforced solely by the Federal Trade


Commission. [17] Source: CMA 1289 1-29
The basic purpose of the securities laws of the United
C. Are enforced by the Federal Trade Commission States is to regulate the issue of investment securities by
and the Justice Department.
A. Providing a regulatory framework in those states
D. Were repealed by an Act of Congress in 1982. that do not have their own securities laws.

B. Requiring disclosure of all relevant facts so that


[13] Source: CMA 1287 1-20
In regulating intrastate local utilities, state public utility investors can make informed decisions.
commissions
C. Prohibiting the issuance of securities that the
A. Usually approve rate schedules that set the same Securities and Exchange Commission determines are
rates for residential, commercial, and industrial not of investment grade.
customers.
D. Channeling investment funds into uses which are
B. Use only original cost methods of rate base economically most important.
valuation.

C. Permit utilities to refuse service to some customers [18] Source: CMA 1289 1-30
within their jurisdiction. Which one of the following statements best describes how
regulatory agencies of the U.S. government are restricted in
D. Usually set rates of return for utilities based on the the adoption of specific regulations?
cost of capital as reflected in the capital markets.
A. Regulations must be consistent with standards
established in the congressional act which created the
[14] Source: CMA 1287 1-21 agency.
Commission (EEOC) is
B. All proposed regulations must be cleared with a
central coordinating agency, the General Accounting A. To restrict enforcement of the Equal Employment
Office. Opportunity Act of 1972 to relatively small
companies in order to minimize disruptions.
C. The agencies must present proposed regulations
to all affected parties for comment. B. To rank the goal of employee productivity ahead
of the goal of equal employment opportunity
D. Businesses subject to the regulation must be considerations.
notified 1 year before the regulation will be put into
effect. C. To avoid legal remedies to achieve the goals of the
EEOC.

[19] Source: CMA 0690 1-2 D. To have businesses achieve employment mixes
The Consumer Product Safety Commission (CPSC) has reflecting the local labor pool of protected groups.
been called by its critics "the most powerful regulatory
agency in Washington." The most likely reason for this
concern is that the CPSC [23] Source: CMA 0691 1-24
State laws that make it illegal to require union membership
A. Covers all consumer products. as a condition for getting or holding a job are known as

B. May ban the production and sale of a product until A. Fair trade laws.
it has formulated a standard.
B. Fair employment laws.
C. Does not permit an industry group to adopt a
voluntary standard. C. Open shop laws.

D. May order the recall of products it has determined D. Right-to-work laws.


to be unsafe.

[24] Source: CMA 0691 1-30


[20] Source: CMA 0690 1-5 An industry that has not experienced significant
Truth-in-lending is one form of price standardization that, deregulation is the
since the adoption of the Consumer Credit Protection Act
on July 1, 1969, has been provided by U.S. government
regulations. The purpose of this legislation is to A. Airline industry.

A. Regulate the amount of interest that may be B. Banking industry.


charged.
C. Trucking industry.
B. Prohibit the use of appraisal fees.
D. Steel industry.
C. Allow immediate wage garnishment by creditors.

D. Disclose the finance charge and the annual [25] Source: CMA 1291 1-19
percentage rate. Over the years, various groups and activities have been
exempted from coverage by the U.S. antitrust laws. Which
one of the following statements regarding such exemptions
[21] Source: CMA 0690 1-6 is correct?
Equal employment opportunity is concerned with, among
other issues, the recruitment and selection of employees. A A. All sales and purchases of products by American
U.S. Supreme Court ruling in 1971 involved the use of firms outside of the U.S. are automatically exempt
pre-employment tests (Griggs v. Duke Power Co.). In that from coverage by the antitrust laws.
case, which was determined under Title VII of the Civil
Rights Act of 1964, the Court ruled that pre-employment B. All resale price maintenance agreements between
tests manufacturers and retailers selling the manufacturers'
product brands are exempt from the federal antitrust
A. Given to minorities, regardless of the reason given, laws.
violate Title VII.
C. Under certain conditions, a person who wishes to
B. Are legal, as long as there is no intent to form an export trading company may obtain a
discriminate. certificate of antitrust immunity from the Commerce
Department after concurrence by the Justice
C. In any form are illegal. Department.

D. Must be directly related to job requirements. D. Water common carriers in foreign trade may enter
into price fixing and market sharing treaties or
agreements provided the agreements are ratified by
[22] Source: CMA 0691 1-21 the U.S. Senate before being put into effect.
A major policy of the Equal Employment Opportunity
The general approach to regulation by the Consumer
[26] Source: CMA 1291 1-20 Product Safety Commission is to
The Environmental Protection Agency (EPA) might control
pollution by setting effluent standards for maximum A. Levy fines against the producers of unsafe
discharge or by constructing a sliding-tax charge based on products.
the amount of effluent emitted. One advantage of the
sliding-tax charge is that B. Set safety standards for various products.

A. It will totally eliminate pollution. C. Increase the freedom of consumer choice.

B. There is greater business incentive to discover new D. Reduce the cost of regulated products.
methods of controlling pollution.

C. It increases the necessity for the EPA to fully [31] Source: CMA 1291 1-26
understand pollution control technology. Social regulation, as exemplified by the Occupational
Safety and Health Act and by the Environmental Protection
D. It increases the degree of direct government Act, is frequently alleged to be inefficient. The perception
intervention in the economy. by firms is that this inefficiency appears to be a result of

A. Concern for the quality of life.


[27] Source: CMA 1291 1-21
With respect to the federal antitrust laws, regulated B. Lack of concern for the magnitude of marginal
industries are benefits relative to marginal costs.

A. Completely exempt. C. The use of flexible rather than rigid standards by


bureaucrats.
B. Covered to the same extent as private industries.
D. Low compliance costs to business firms.
C. Covered to the extent determined by the
applicable regulatory agency.
[32] Source: CMA 1291 1-28
D. Covered to the extent determined by statute and The National Labor Relations Act, also known as the
the courts. Wagner Act, guarantees labor certain rights. This Act

A. Requires management to make detailed financial


[28] Source: CMA 1291 1-22 reports to the National Labor Relations Board.
Federal regulatory agencies do not have power to
B. Designates a list of unfair labor practices on the
A. Impose agency taxes on private industry. part of unions.

B. Issue rules and regulations. C. Guarantees the right of self-organization and the
right to collective bargaining with management.
C. Investigate violations of statutes and rules.
D. Requires that elections of union officers be
D. Recommend penalties for violations of statutes regularly scheduled and that secret ballots be used.
and rules.

[33] Source: CMA 1291 1-29


[29] Source: CMA 1291 1-24 The Taft-Hartley Act attempted to restore balance
Violations of federal regulations on equal employment between the rights and obligations of employees and those
opportunity are sometimes inferred from the fact that a firm of employers by
has a very small percentage of protected groups among its
employees. These regulations are sometimes attacked on A. Permitting strikes in cases of health and safety
the grounds that they may be in conflict with the economic violations.
concept that
B. Outlawing the union shop.
A. The applicant who would be most productive for
the job should be hired. C. Designating a list of unfair labor practices on the
part of employers.
B. The work force employed should be in the same
proportion as the demographics of the total D. Designating a list of unfair labor practices on the
population. part of unions.

C. Equal opportunity strengthens the seniority system.


[34] Source: CMA 1291 1-30
D. Wages are inversely related to marginal The concept of comparable worth has become an
productivity. important issue in the U.S. in recent years. This concept
addresses the comparability of

[30] Source: CMA 1291 1-25 A. U.S. wage rates with foreign wage rates adjusted
for differences in exchange rates.
C. Compliance costs are a burden for small firms,
B. Wage rates for different jobs requiring different and Congress has chosen to exempt such firms.
skills.
D. The lobby for small businesses has been so active
C. Wage rates for the same job in union and that Congress has chosen to exempt such firms.
non-union environments.

D. Wage rates for the same job in the government [39] Source: CMA 0693 1-20
and the private sector. The Clean Air Act of 1970 charged the Environmental
Protection Agency (EPA) with all of the following
responsibilities except
[35] Source: CMA 0693 1-11
The basic purpose of the Securities Exchange Act of 1934 A. Setting standards without consideration for the
was to cost of compliance to manufacturers.

A. Regulate outstanding securities.


B. Nullifying the right of an individual citizen to bring
B. Regulate new issues of securities. suit to enforce standards as such suits will now be
initiated by the EPA.
C. Protect investors from investment losses.
C. Setting standards to prevent damage to items such
D. Ensure the public that stock market crashes like as visibility, crops, and buildings.
that of 1929 would not occur again.
D. Establishing minimum ambient air standards for the
entire country.
[36] Source: CMA 0693 1-14
The private placement of debt securities has all of the
following advantages over offerings to the general public [40] Source: CMA 0693 1-21
except that a private placement The federal Consumer Product Safety Commission
(CPSC) was created in 1972 and administers an act
A. Does not require Securities and Exchange covering those consumer products not already regulated by
Commission regulation. the federal government. Specifically, Section 15 of the Act
(CPSA) requires manufacturers to take corrective steps for
B. Can be completed in less time than public a product that fails to comply with a product safety rule or
offerings. contains a defect that could create a substantial product
hazard. This provision is included because
C. Has no flotation expenses.
A. The CPSC is dependent on the manufacturers to
D. Can include indentures that are "tailor-made" to provide information on product safety.
the borrower's needs.
B. Voluntary standards are preferred to compulsory
ones, and this pushes regulation back into the hands
[37] Source: CMA 0693 1-13 of manufacturers.
Blue-sky laws are
C. It encourages the CPSC to apply cost-benefit
A. The detailed regulations of the Securities and analysis in formulating and implementing consumer
Exchange Commission designed to prevent fraudulent product safety standards.
or misleading security issues.
D. The manufacturer may be compelled to publicize
B. Regulations of the Securities and Exchange this defect to consumers and/or refund the purchase
Commission governing the treatment of goodwill in price.
the sale of securities.

C. State laws designed to prevent fraudulent or [41] Source: CMA 0693 1-22
misleading security issues. In recent years, a number of consumer groups have used a
variety of methods to persuade companies to change
D. Local laws regulating the issuance of securities for behavior. Which one of the following weapons has not
companies whose operations affect the environment. been used by consumer groups in recent years?

A. Boycotts or refusals to purchase from the


[38] Source: CMA 0693 1-19 offending company.
Many social regulations exempt small business from their
application. This exemption has been enacted because B. Shareholder resolutions at annual shareholder
meetings to direct a company's board of directors to
A. Small businesses have no significant impact on do or not to do certain things.
jobs and the environment.
C. Creation of a competing company to replace the
B. There are too many small businesses, and funds offending company.
provided are inadequate to regulate them effectively.
D. Cooperation through meetings that attempt to find
common ground for action by both parties. B. Market allocation or division of customers,
markets, or production.

[42] Source: CMA 0693 1-23 C. Concerted refusals to deal or boycotts involving
Which one of the following is not a characteristic of the any third party.
Occupational Safety and Health Administration (OSHA)?
OSHA D. Tie-in sales or any effort to force a buyer to
purchase less desirable products in order to purchase
A. Encourages labor-management committees to the desired product.
formulate safety and health programs.

B. Inspections are primarily focused on health issues,


including long-term exposure to such substances as [46] Source: CMA 0693 1-27
asbestos and cotton dust. The impact of successful prosecution for restraint of trade
violations of the Sherman Antitrust Act has been less than
C. Has the authority to levy monetary penalties on many proponents hoped for because
non-compliant employers.
A. Only a limited number of criminal prosecutions
D. Inspections are primarily reactionary; that is, take have ever been filed under Section 1 of the Act.
place after major injuries have occurred.
B. Most companies avoid behaviors that might be
construed as antitrust violations because of the
[43] Source: CMA 0693 1-24 likelihood of fines and even jail sentences.
Regulated industries, particularly those subject to
industry-specific economic regulation, have several C. The predominant criterion for criminal prosecution
common characteristics. Which one of the following is market share; a company's intent is inconsequential.
characteristics is not associated with most regulated
industries? D. The courts created in the beginning a "rule of
reason" that required determining whether
A. These industries are capital intensive. competition was lessened by the company's behavior.

B. The social side effects or consequences of the


actions of these firms may be undesirable. [47] Source: CMA 0693 1-28
The first major federal equal job opportunity law since the
C. Such industries are often described as "affected 1970s is the Americans with Disabilities Act (ADA) of
with a public interest" or considered vital industries. 1990. This act does not

D. Most of these industries are permitted to earn a A. Ban discrimination against people with disabilities
return that is equitable to all consumers. in employment.

B. Provide tax incentives for compliance costs.


[44] Source: CMA 0693 1-26
Antitrust suits, particularly civil rather than criminal suits, C. Ban discrimination against people with disabilities
may be brought by many different individuals or agencies. in transportation.
Which one of the following does not have power to bring
such suits. D. Provide federal funds for the implementation of
ADA by employers and those providing public
A. U.S. Department of Justice. accommodation.

B. A state attorney general, when authorized by a


state antitrust law. [48] Source: CMA 0693 1-29
In recent years, governmental bodies have tried a number
C. A private party, such as a competitor. of devices to encourage environmental stewardship. At
least fourteen major federal acts or amendments to acts
D. U.S. Department of Commerce. were passed between 1969 and 1990. Which one of the
following is not a governmental device used in recent
years?
[45] Source: CMA 0693 1-25
Supreme Court interpretations of the Sherman Act of 1890 A. Direct regulation or legally enforceable
have determined that certain types of agreements, environmental quality standards.
conspiracies, or combinations are in and of themselves so
restrictive of competition as to be conclusively presumed B. Pollution charges or fees for undesirable waste
unreasonable restraints of trade. These "per se" violations released by the firm.
do not involve the application of a "rule of reason" but are
considered illegal merely because they exist. Which one of C. Relocation of facilities to offshore sites where
the following offenses is the most serious, i.e., which one governmental standards are less stringent.
has produced the most criminal antitrust violations?
D. Buying and selling of pollution rights involving
A. Price-fixing or tampering with the price structure. companies whose emissions are below the standard.
A. Make pollution illegal.
[49] Source: CMA 0693 1-30
Pharmaceutical companies must be in compliance with B. Regulate the amount of pollution a firm can create.
regulations set forth by the Food and Drug Administration
(FDA). The FDA is responsible for all of the following C. Promote social awareness about the harm due to
except pollution.

A. Allowing extensions to patent lives of D. Allow marketable permits for pollution that firms
pharmaceutical companies in order to recover time can buy or sell like any other commodity.
lost during the premarket FDA regulatory review.

B. Considering benefit-risk trade-offs when [54] Source: CMA 1293 1-6


evaluating the safety of drugs prior to approval. A horizontal merger is a merger between

C. Maintaining purity standards and regulations A. Two or more firms from different and unrelated
concerning drug composition and potency. markets.

D. Approving drugs that lack substantial evidence B. Two or more firms at different stages of the
that they will have the effects that are represented in production process.
advertisements.
C. A producer and its supplier.

[50] Source: CMA 1293 1-1 D. Two or more firms in the same market.
The Sherman Antitrust Act

A. Is intended for international corporations. [55] Source: CMA 1293 1-7


In unionized and nonunionized sectors of the economy,
B. Prohibits labor unions from forming closed shops. labor unions generally

C. Outlaws all monopolies. A. Result in wage equality in both sectors.

D. Discourages firms from monopolizing. B. Raise wages in the unionized sector.

C. Raise wages at the same rate in both sectors.


[51] Source: CMA 1293 1-2
In a regulated industry such as electricity or gas, D. Lower wages in both sectors.

A. The forces of demand and supply determine price.


[56] Source: CMA 1293 1-8
B. Firms can enter or exit the industry easily. In relation to labor unions, a closed-shop is where

C. Firms can decide on the quality of product or A. Only those belonging to the union can work.
service.
B. Belonging to the union is not necessary.
D. Regulatory bodies determine the quality and set
the price. C. Collective bargaining is not permitted.

D. Workers have the right to work.


[52] Source: CMA 1293 1-4
The primary reason for social regulation (Clean Air Act,
Water Pollution Control Act, Food and Drug Act, etc.) is [57] Source: CMA 1293 1-9
that The most common form of business organization in terms of
number of firms in the United States is the
A. The free market provides minimal safety and
environmental protection. A. Partnership.

B. Consumer and environmental groups are politically B. Limited partnership.


powerful.
C. Sole proprietorship.
C. Social benefits from such regulations always
exceed the costs. D. Corporation.

D. Social regulation is more desirable.


[58] Source: CMA 1293 1-11
Environmental policies undertaken in the United States that
[53] Source: CMA 1293 1-5 attempt to weigh the costs and benefits of environmental
An efficient way to control pollution in line with the improvement
expectations of the Clean Air Act Amendments of 1990 is
to A. Will result in the elimination of all pollution.
A. Establish set profit percentages for firms in
B. Cannot work because business firms are not regulated industries.
willing to cooperate.
B. Prohibit firms in the same industry from engaging in
joint ventures.
C. Grant tax relief to the firms who pollute the least.
C. Ensure a free and competitive market in which
D. Would leave an acceptable amount of pollution. consumer demand dictates prices.

D. Limit competition to increase output and lower


[59] Source: CMA 1293 1-12 prices.
Commercial bank deposits are currently insured to
$100,000 per account by the
[63] Source: Publisher
A. Federal Reserve System. Pricing agreements are allowable in which of the following
situations?
B. Federal Saving and Loan Insurance Corporation.
A. An agreement to lower prices for consumers.
C. Federal Deposit Insurance Corporation.
B. An agreement to fix prices to eliminate unfair
D. U.S. Treasury. competition.

C. An agreement to stabilize prices reasonably and


[60] Source: Publisher fairly for consumers.
Which of the following statements about the National
Environmental Policy Act (NEPA) is most likely to be D. Never.
false?

A. NEPA requires federal agencies to consider [64] Source: Publisher


certain potential environmental consequences in their To be a monopoly, a firm must
decision-making process.
A. Be the sole provider of the product.
B. NEPA allows the federal government to bring suit
against any private person who violates NEPA's B. Have a market share of at least 80%.
provisions.
C. Have the power to control prices or exclude
C. Under NEPA, federal agencies do not have to competition.
give environmental considerations priority over other
concerns in their decision-making processes. D. Have entered into contracts, combinations, or
conspiracies in restraint of trade.
D. NEPA augments the power of every existing
agency with respect to considering the environmental
consequences of their proposed actions. [65] Source: Publisher
Which of the following firms are subject to the requirements
of the Foreign Corrupt Practices Act?
[61] Source: Publisher
Which of the following statements is true with respect to the A. All corporations that engage in intrastate or
Clean Water Act (CWA)? interstate commerce.

A. Despite the CWA's prohibitions, it allows persons B. All corporations required to be registered under
to discharge pollutants into waters subject to its the Securities Act of 1933.
jurisdiction as long as navigation thereon will not be
permanently obstructed. C. All corporations required to be registered under
the Securities Exchange Act of 1934.
B. The CWA subjects all bodies of water located in
the United States, whether flowing or not, to its D. All corporations incorporated in the U.S.
protection.

C. The notion of protecting waters within the [66] Source: Publisher


jurisdiction of the United States began with the A major result of the Foreign Corrupt Practices Act is that
CWA. corporations are now required to

D. The CWA seeks to restore and maintain the A. Keep accurate accounting records and maintain
physical and biological integrity of the waters of the an internal control structure.
United States.
B. Prepare financial statements in accordance with
U.S. and international accounting standards.
[62] Source: Publisher
Antitrust laws are intended to C. Produce information to the SEC on foreign
commerce and foreign political party information.
business.
D. Adhere to U.S. auditing standards.

[72] Source: CMA 0694 1-16


[67] Source: Publisher The Sherman Antitrust Act of 1890 prohibits
The Foreign Corrupt Practices Act prohibits
A. Price discrimination, tying contracts,
A. Bribes to all foreigners. anti-competitive mergers, and interlocking
directorates.
B. Small bribes to foreign officials that serve as
facilitating or grease payments. B. Unfair competition and deceptive business
practices.
C. Bribery only by corporations and their
representatives. C. Mergers without prior notification to the Justice
Department.
D. Bribes to foreign officials to influence official acts.
D. Restraint of trade and monopoly.

[68] Source: CIA 0594 IV-67


Which of the following is not considered to be an [73] Source: CMA 0694 1-17
advantage of organizing a business as a sole The Taft-Hartley Act of 1947 was an attempt to restore
proprietorship? balance between the rights of employers and those of
employees by
A. Easy and inexpensive to organize.
A. Outlawing the union shop.
B. Allows freedom of action for the entrepreneur.
B. Imposing mandatory binding arbitration on both
C. Provides strong incentives to manage the business employers and unions.
efficiently.
C. Designating certain labor practices on the part of
D. Allows the proprietor to carry out all basic unions as unfair and outlawing the closed shop.
management functions.
D. Permitting strikes on the part of federal
employees.
[69] Source: CIA 0594 IV-66
In which legal form of business organization do the owners
of the business enjoy limited liability? [74] Source: CMA 0694 1-18
The Robinson-Patman Act of 1936 prohibits price
A. Sole proprietorship. discrimination unless

B. Partnership. A. The price is set to meet a competitor's price


quote.
C. Corporation.
B. The price reflects cost increases caused by
D. Monopoly. government specification.

C. The price is set in a market characterized by


[70] Source: CIA 0589 IV-47 monopolistic competition.
Which of the following represents a disadvantage of the
partnership form of business organization? D. Specific prior approval is obtained from the
Justice Department.
A. Unlimited liability.

B. Double taxation. [75] Source: CMA 1294 1-10


Which one of the following antitrust laws prohibits price
C. Complexity in formation. discrimination, tying contracts, anticompetitive mergers, and
interlocking directorates?
D. Regulatory requirements.
A. Sherman Antitrust Act.

[71] Source: CIA 0593 IV-47 B. Clayton Antitrust Act.


A limited partnership is typically used to limit the
C. Antitrust Improvements Act.
A. Withdrawals of individual partners.
D. Federal Trade Commission Act.
B. Duration of the partnership.

C. Liabilities of some of the partners. [76] Source: CMA 1294 1-11


The National Labor Relations Act of 1935, also known as
D. Authority of general partners to manage the the Wagner Act,
A. Provides for compulsory binding arbitration in the C. Covered as determined by the Department of
case of national emergency. Justice.

B. Guarantees the right of self-organization and the D. Covered to the same extent as any other industry.
right to collective bargaining with management.

C. Designates a list of unfair labor practices on the [81] Source: CMA 1295 1-21
part of unions. Social regulation is often criticized by industry as inefficient.

D. Requires that union officials be elected on a Firms perceive this inefficiency to be a result of
regular basis and that secret ballots be used.
A. The failure to consider the marginal benefits
relative to the marginal costs.
[77] Source: CMA 1294 1-12
The Labor-Management Relations Act, also known as the B. Lenient enforcement policies.
Taft-Hartley Act of 1947,
C. Concern for the quality of life but not the quality of
A. Prohibits employers from interfering with the right products.
of employees to form unions.
D. The use of the internal revenue tax code instead of
B. Obligates employers to bargain in good faith with strict compliance penalties.
unions.

C. Outlaws yellow-dog contracts, which require [82] Source: CMA 1295 1-22
employees to remain as nonunion members. Which one of the following transactions would be
considered a violation of the Robinson-Patman Act?
D. Prohibits unions from coercing employees to
become union members. A. The sale of goods of like quality at different prices
to two different wholesalers, both of whom are
located outside the United States.
[78] Source: CMA 1294 1-13
The Sherman Antitrust Act B. The sale of goods of like quality within the United
States at different prices based on cost differences
A. Created the Federal Trade Commission. related to the method of delivery.

B. Did not establish penalties for offenders of the C. The sale of goods of like quality within the United
Act. States at different prices to two different wholesalers;
all parties are located within the same state.
C. Prohibits mergers and acquisitions that
substantially lessen competition. D. The sale of goods of like quality within the United
States but across state lines at different prices to two
D. Declares illegal, attempts at restraint of trade. different wholesalers in the same geographic area.

[79] Source: CMA 1294 1-14 [83] Source: CMA 1295 1-23
Social legislation, as exemplified by the Occupational Which one of the following examples of corporate behavior
Safety and Health Act and Environmental Protection Act, is would most clearly represent a violation of the Sherman
frequently criticized for being inefficient because the Act?
agencies
A. A retailer offers quantity discounts to large
A. Use flexible rather than rigid standards. institutional buyers.

B. Impose compliance costs that are low compared B. The members of a labor union meet and agree not
to the benefits received by society. to work for a specific firm unless the starting wage is
at least $10 per hour.
C. Rely heavily on the free market to allocate
resources. C. Two firms that are in different, unrelated industries
merge.
D. Rarely consider the marginal benefits relative to
the marginal costs. D. Two firms in the same industry agree in a
telephone conversation to submit identical bids on a
government contract.
[80] Source: CMA 1295 1-20
With respect to federal antitrust laws, regulated industries
are [84] Source: CMA 1295 1-24
The Clayton Act, as amended, prohibits all of the following
A. Completely exempt. except

B. Covered as determined by statute and the courts. A. Tying contracts that require a customer who is
buying one product to buy a related but perhaps II. National Highway Traffic Safety Administration.
unwanted product. III. Consumer Product Safety Commission.
IV. Occupational Safety and Health Administration.
B. Price discrimination by sellers. V. Environmental Protection Agency.
Which of the above agencies are considered social
C. Interlocking directorates in large competing agencies?
organizations.
A. I, II, and IV only.
D. Unfair and deceptive business practices, such as
misleading advertising. B. I and II only.

C. I, II, and III only.


[85] Source: CMA 1295 1-25
The acquisition of a retail shoe store by a shoe D. All of the agencies.
manufacturer is an example of

A. Vertical integration. [90] Source: CMA 1295 1-30


The two major functions of the Federal Trade Commission
B. A conglomerate. are

C. Market extension. A. Antitrust actions and the regulation of foreign


trade.
D. Product extension.
B. Import quality inspections and anti-dumping
measures.
[86] Source: CMA 1295 1-26
Blue sky laws are
C. Antitrust actions and consumer protection.
A. Federal laws that make it unlawful to use
deceptive practices in the sale of securities. D. Price discrimination and unfair trade practices.

B. Federal laws that limit the amount of air pollution


in a specific geographic area. [91] Source: CMA 0685 1-34
Membership in labor unions as a proportion of the total
C. State laws that regulate the sale of securities. work force has declined in the past decade. The reason
most often given for this decline is that
D. State laws that regulate the environment.
A. The expansion in employment has occurred
primarily in service industries.
[87] Source: CMA 1295 1-27
Which one of the following federal acts requires unions to B. Unions have been less aggressive in organizing.
retain financial records and submit financial reports to
federal authorities? C. Workers have become disenchanted with union
leaders.
A. Taft-Hartley Act of 1947.
D. A larger proportion of union members have
B. Wagner Act of 1935. reached retirement age and the replacement workers
are not joining the union.
C. Securities Exchange Act of 1934.

D. Landrum-Griffin Act of 1959. [92] Source: CMA 0685 1-35


Business organizations find that union behavior is changing
due in part to the change in composition of union
[88] Source: CMA 1295 1-28 membership. A change that is not occurring in union
Which one of the following is not exempted from federal membership is that on the average there are more
antitrust regulation?
A. Members from minority groups.
A. Labor unions.
B. Women members.
B. Intrastate commerce.
C. Members from blue-collar jobs.
C. Patents and copyrights.
D. Members from the public sector.
D. Telecommunications companies.

[93] Source: Publisher


[89] Source: CMA 1295 1-29 Corporate social responsibility is
Listed below are five federal agencies.
A. Effectively enforced through the controls
I. Food and Drug Administration. envisioned by classical economics.
B. Defined as the obligation to shareholders to earn a A. Outlaws interlocking directorates in large
profit. competing corporations.

C. More than the obligation to shareholders to earn a B. Prohibits price discrimination by sellers.
profit.
C. Forbids tying contracts that require a purchaser to
D. Defined as the obligation to serve long-term, buy an unwanted product to get another.
organizational interests.
D. Forbids unfair and deceptive business practices,
such as misleading advertising.
[94] Source: Publisher
A common argument against corporate involvement in
socially responsible behavior is that [99] Source: CMA 0696 1-24
If a product is believed to pose an imminent hazard to
A. It encourages government intrusion in decision consumers, the Consumer Product Safety Commission
making. (CPSC)

B. As a legal person, a corporation is accountable for A. May seek a court order to have the product
its conduct. banned or seized.

C. It creates goodwill. B. May ban or seize the product upon a majority


vote by the commissioners of the CPSC.
D. In a competitive market, such behavior incurs
costs that place the company at a disadvantage. C. May ban or seize the product, but only after a
notice has been published in the Federal Register for
30 days.
[95] Source: CMA 0696 1-18
Resale price maintenance is an example of D. Can only ask the manufacturer voluntarily to
withdraw or recall the product.
A. Horizontal price fixing.

B. Vertical price fixing. [100] Source: CMA 0696 1-25


Which one of the following questions can be asked by an
C. Preemptive buying. employer during an employment interview ensuring
compliance with anti-discrimination rules?
D. Tying arrangements.
A. "How old are you?"

[96] Source: CMA 0696 1-21 B. "Do you have a disability?"


A main provision of the Securities Act of 1933, as
amended in 1934, is the requirement that C. "Are you married?"

A. Bonds be issued only under a trust indenture D. "Do you have references?"
approved by the Securities and Exchange
Commission (SEC).
[101] Source: CMA 0696 1-26
B. Public utility holding companies register with the A factory moves from a non-right-to-work state to a
SEC. right-to-work state, and the union continues to represent
the employees. In the right-to-work state,
C. New securities offered for sale in interstate
commerce be registered with the SEC. A. The employees can be forced to join the union
before they start to work.
D. All security brokers be licensed by the SEC.
B. The employees can be forced to join the union
after thirty days on the job.
[97] Source: CMA 0696 1-22
The Clayton Act of 1914 prohibits C. The employees can decide not to be involved with
the union.
A. Closed-shop labor unions.
D. The employees can be forced to make payments
B. Sellers' price discrimination. to the union.

C. Group boycotts.
[102] Source: CMA 0696 1-27
D. Oligopolies. Which one of the following approaches to regulation best
describes mandatory labeling information concerning food
content and nutritional values?
[98] Source: CMA 0696 1-23
The Federal Trade Commission Act, as amended, A. Unregulated markets subject to antitrust
enforcement.
A. Requires pre-approval from the Federal Trade
B. Disclosure of all relevant information. Commission for interlocking directorates.

C. Changes in liability rules toward a stricter liability. B. Prohibits price discrimination.

D. Creation of marketable property rights. C. Requires notification prior to a merger.

D. Prohibits restraints of trade and monopoly.


[103] Source: CMA 0696 1-28
Which one of the following forms of law is created when
regulatory agencies transform statutes into regulations and [108] Source: CMA 1296 1-24
enforcement procedures? Sunset laws refer to

A. Constitutional law. A. Protection of employees nearing retirement.

B. Statutory law. B. Federal laws that prohibit the sale of securities for
companies that have not been in business for at least
C. Administrative law. 3 months.

D. Judicial law. C. State laws that prohibit the sale of alcohol on


Sunday.

[104] Source: CMA 1296 1-20 D. A periodic review of, and a fixed termination date
A franchisor's requirement that its franchisees buy inputs for, government programs.
from a particular supplier is

A. Illegal according to the provisions of the [109] Source: CMA 1296 1-25
Robinson-Patman Act. All of the following are legal rights of shareholders in U.S.
publicly traded companies except the right to
B. Illegal according to the principles of common law.
A. Vote on major mergers and acquisitions.
C. Legal, but the franchisor must receive approval
from the Federal Trade Commission. B. Receive dividends if declared.

D. Legal as long as such a requirement is necessary C. Vote on charter and bylaw changes.
to assure product quality.
D. Vote on major management changes.

[105] Source: CMA 1296 1-21


A basic purpose of the securities laws in the United States [110] Source: CMA 1296 1-29
is to regulate the issuance of investment securities by The Americans with Disabilities Act (ADA) of 1990

A. Requiring disclosure of all relevant information so A. Permits employers to inquire about a job
that investors can make informed decisions. applicant's prior health insurance claims.

B. Prohibiting the issuance of non-investment grade B. Bans discrimination against employees with
securities. physical disabilities, but not those with mental
disabilities.
C. Ensuring that all shareholders have an equal vote
in the election of a board of directors. C. Requires organizations with 25 or more

D. Providing a regulatory framework for those states employees to provide reasonable accommodation for
that do not have their own securities laws. employees with disabilities.

D. Provides federal funds to employers who


[106] Source: CMA 1296 1-22 implement its provisions.
The Sherman Antitrust Act

A. Established the Federal Trade Commission. [111] Source: CMA 1296 1-30
The Insider Trading and Securities Fraud Enforcement Act
B. Prohibits collective boycotts. of 1988 in conjunction with the Insider Trading Sanctions
Act of 1984
C. Prohibits price discrimination.
A. Enacted monetary penalties for insider trading for
D. Established the concept of a patent. the first time.

B. Increased monetary penalties for insider trading.


[107] Source: CMA 1296 1-23
The Antitrust Improvements Act of 1976 C. Enacted criminal penalties for market manipulation
and securities fraud. rules.

D. Empowered the Securities and Exchange C. Regulatory rules being rigid and hard to change.
Commission with the authority to recover gains
resulting from the illegal use of insider information. D. Regulations increasing the cost of products and
services.

[112] Source: CMA 0697 1-22


A large public company that is well-known can reduce the [117] Source: CMA 0697 1-29
time required to register and issue securities by using a(n) All the following are functions of the Securities and
Exchange Commission except the
A. Shelf registration.
A. Review of stock trades by corporate insiders.
B. Indenture agreement.
B. Regulation of interstate offerings of new securities
C. Secondary market registration. to the public.

D. Red herring registration. C. Setting of rules concerning the proxy process of


large public companies.

[113] Source: CMA 0697 1-25 D. Determination of fair trading prices for the
Which one of the following antitrust laws prohibits price common stock of large public companies.
discrimination and other exclusionary practices that may
give certain firms a competitive advantage over other firms
in the same market? [118] Source: Publisher
The Comprehensive Environmental Response,
A. Celler-Kefauver Act. Compensation, and Liability Act (CERCLA), also
generically known as the "Superfund," applies to the release
B. Federal Trade Commission Act. of "hazardous substances." Which of the following is
statutorily included in the definition of the term "hazardous
C. Sherman Act. substance"?

D. Robinson-Patman Act. A. Crude oil.

B. Gasoline.
[114] Source: CMA 0697 1-26
Which one of the following laws addresses the issue of C. Asbestos.
insider trading?
D. Natural gas.
A. Federal Trade Commission Act.

B. Securities Exchange Act. [119] Source: Publisher


Which of the following statements about the National
C. Clayton Act. Environmental Policy Act (NEPA) is most likely to be
incorrect?
D. North American Free Trade Agreement.
A. NEPA requires federal agencies to consider
environmental consequences in their decision-making
[115] Source: CMA 0697 1-27 process.
Airlines headquartered in the United States have
experienced deregulation in many aspects of their business. B. NEPA allows the federal government to bring suit
Which one of the following aspects of the business is still against any private person who violates NEPA's
subject to regulation by the federal government? provisions.

A. On-time departure standards. C. Under NEPA, federal agencies do not have to


give environmental considerations priority over other
B. Domestic fare schedules. concerns in their decision-making processes.

C. Collective bargaining. D. NEPA augments the power of existing agencies


with respect to considering environmental
D. International fare schedules. consequences of proposed actions.

[116] Source: CMA 0697 1-28 [120] Source: Publisher


All of the following are criticisms generally made of federal Which of the following statements regarding the Clean
regulatory agencies and policies except Water Act (CWA) is correct?

A. Taxes imposed by agencies being too high. A. It allows persons to discharge pollutants into
waters subject to its jurisdiction as long as navigation
B. Political considerations influencing policies and thereon will not be permanently obstructed.
making.
B. The CWA subjects all bodies of water located in
the United States, whether flowing or not, to its B. As a legal person, a corporation is accountable for
protection. its conduct.

C. The notion of protecting waters within the C. It creates goodwill.


jurisdiction of the United States began with the
CWA. D. In a competitive market, such behavior incurs
costs that place the company at a disadvantage.
D. The CWA seeks to restore and maintain the
physical and biological integrity of the waters of the
United States. [125] Source: Publisher
Integrity is an ethical requirement for all financial
managers/management accountants. One aspect of integrity
[121] Source: CMA 1294 1-9 requires
(Refer to Figure 4.) One way of measuring government
performance with respect to antitrust regulation is A. Performance of professional duties in accordance
cost-benefit analysis. In the graph, P1 and Q1 represent with applicable laws.
the price and quantity under a monopoly. Effective
government regulation could lower the price to P2 which B. Avoidance of conflict of interest.
could increase the quantity to Q2. On the graph, the area
ABC represents the C. Refraining from improper use of inside
information.
A. Total cost of producing the additional output.
D. Maintenance of an appropriate level of
B. Excess unsatisfied demand caused by the professional competence.
regulation.

C. Efficiency gain from antitrust action. [126] Source: Publisher


Under the express terms of the IMA Code of Ethics, a
D. Total benefit generated by the change. financial manager/management accountant may not

A. Advertise.
[122] Source: Publisher
Sheila is a financial manager who has discovered that her B. Encroach on the practice of another financial
company is violating environmental regulations. If her manager/management accountant.
immediate superior is involved, her appropriate action is to
C. Disclose confidential information unless authorized
A. Do nothing since she has a duty of loyalty to the or legally obligated.
organization.
D. Accept other employment while serving as a
B. Consult the audit committee. financial manager/management accountant.

C. Present the matter to the next higher managerial


level. [127] Source: CMA 1
According to Statements on Management Accounting
D. Confront her immediate superior. Number 1C (SMA 1C) (revised), Standards of Ethical
Conduct for Practitioners of Management Accounting and
Financial Management, a practitioner has a responsibility to
[123] Source: Publisher recognize professional limitations. Under which standard of
Corporate social responsibility is ethical conduct would this responsibility be included?

A. Effectively enforced through the controls A. Competency.


envisioned by classical economics.
B. Confidentiality.
B. Defined as the obligation to shareholders to earn a
profit. C. Integrity.

C. More than the obligation to shareholders to earn a D. Objectivity.


profit.

D. Defined as the obligation to serve long-term, [128] Source: CMA 2


organizational interests. At Key Enterprises, the controller is responsible for
directing the budgeting process. In this role, the controller
has significant influence with executive management as
[124] Source: Publisher individual department budgets are modified and approved.
A common argument against corporate involvement in For the current year, the controller was instrumental in the
socially responsible behavior is that approval of a particular line manager's budget without
modification, even though significant reductions were made
A. It encourages government intrusion in decision to the budgets submitted by other line managers. As a
token of appreciation, the line manager in question has
given the controller a gift certificate for a popular local
restaurant. In considering whether or not to accept the
certificate, the controller should refer to which section of
Statements on Management Accounting Number 1C
(SMA 1C) (revised), Standards of Ethical Conduct for
Practitioners of Management Accounting and Financial
Management?

A. Competency.

B. Confidentiality.

C. Integrity.

D. Objectivity.
Answer (C) is incorrect because direct corporate
CMA PART 1 D contributions are not allowed. However,
Domestic Institutional corporations, trade associations, and unions may
form PACs and encourage employees to participate,
Environment of Business and company funds may be used to organize and
administer the PAC.
ANSWERS
Answer (D) is incorrect because individuals may give
funds either directly to candidates or to PACS.

[1] Source: CMA 0685 1-31


[4] Source: CMA 1286 1-26
Answer (A) is incorrect because it is not meaningful
in this context. Answer (A) is incorrect because the concept of
negligence requires proof of fault.
Answer (B) is incorrect because it is not meaningful in
this context. Answer (B) is incorrect because the theory of strict
liability requires neither a warranty nor even privity of
Answer (C) is incorrect because it is not meaningful contract.
in this context.
Answer (C) is incorrect because contributory
Answer (D) is correct. Sunset laws provide for a negligence is not a defense in a strict liability case, but
periodic legislative review of and a fixed termination comparative negligence may be pleaded in some
date for regulatory acts and agency enabling statutes. states.
The legislative body must take affirmative action to
determine a program's usefulness and to reauthorize Answer (D) is correct. Strict liability may be imposed
it. The objective is to avoid the creation of programs in product liability cases although the defendant was
that continue after their purposes have been achieved. not at fault. In a suit against the manufacturer, a
plaintiff must prove that the product was defective,
the defect rendered it unreasonably dangerous to a
[2] Source: CMA 1286 1-23 user, the seller was engaged in the business of selling
the particular product, and the product reached the
Answer (A) is incorrect because all private user without substantial change from the condition in
corporations are created under state statutes since no which it was sold. However, absolute, not strict,
federal incorporation statute exists. liability is imposed for damages caused by inherently
dangerous products.
Answer (B) is correct. Corporations do not exist at
common law. They are entirely creatures of statutory
law. In the U.S., private corporations are chartered [5] Source: CMA 0687 1-13
by the individual state governments.
Answer (A) is incorrect because antitrust laws do not
Answer (C) is incorrect because all private regulate prices. However, prices may not be
corporations are created under state statutes since no discriminatory or collusive.
federal incorporation statute exists.
Answer (B) is incorrect because collecting price
Answer (D) is incorrect because a private information is lawful.
corporation can only come into being if the state
incorporation statute is complied with. The Answer (C) is incorrect because antitrust laws apply
corporation must exist before directors can be to all firms engaged in interstate commerce,
elected. regardless of size.

Answer (D) is correct. The Sherman Antitrust Act of


[3] Source: CMA 1286 1-28 1890 was the first U.S. antitrust law. It prohibited
contracts, combinations, and conspiracies in restraint
Answer (A) is correct. Federal law prohibits business of trade and made monopolizing or attempting to
corporations from donating to political election monopolize any part of interstate commerce a
campaigns at the federal level. Note that this does not criminal offense. Allocating customers among firms
prohibit donations to state election campaigns, within an industry, for example, by division of
although some states have such laws. Political action markets, is a per se violation of the Act.
committees (PACs) have therefore grown in
importance over the past decade as a means of
circumventing this prohibition. [6] Source: CMA 0687 1-15

Answer (B) is incorrect because direct corporate Answer (A) is correct. Tie-in sales involve a
contributions are not allowed. However, seller/lessor's requirement that the buyer/lessee
corporations, trade associations, and unions may purchase/lease another distinct product in order to
form PACs and encourage employees to participate, obtain the first. They are generally prohibited under
and company funds may be used to organize and the Clayton Act of 1914 unless a small company is
administer the PAC. attempting to enter a market or the tie-in is needed to
protect the firm's goodwill (e.g., by maintaining a
certain quality standard). However, this prohibition does not apply if such
contracts are designed to assure product quality.
Answer (B) is incorrect because tie-in sales
requirements are generally illegal if the seller has Answer (B) is incorrect because these contracts are
enough market power to restrict competition. not always illegal, and patents have nothing to do with
these provisions.
Answer (C) is incorrect because tie-in sales
requirements are generally illegal if the seller has Answer (C) is incorrect because price discrimination
enough market power to restrict competition. is prohibited under the Robinson-Patman Act of
1936.
Answer (D) is incorrect because tie-in sales
requirements are generally illegal if the seller has Answer (D) is incorrect because exclusive dealing is
enough market power to restrict competition. generally prohibited under the Clayton Act.

[7] Source: CMA 0687 1-22 [10] Source: CMA 1287 1-17

Answer (A) is incorrect because the FCC allocates Answer (A) is incorrect because profits and prices
access to the radio spectrum to those who will best are not set by antitrust laws other than to the extent
use the medium. that price discrimination is prohibited.

Answer (B) is correct. The FCC is an independent Answer (B) is incorrect because firms may enter into
agency formed in 1934 to regulate all methods of joint ventures with foreign firms.
communications over the public airwaves. The
functions of the FCC include allocating transmission Answer (C) is incorrect because patents are available
frequencies, issuing transmission licenses, fostering to all inventors, regardless of size.
effective and efficient use of communication
resources, and regulating common communications Answer (D) is correct. Antitrust laws are designed to
carriers engaged in interstate and foreign operations. promote more efficient allocation of resources,
Among the rules is a limitation on the number of greater choice for consumers, greater business
television stations that can be owned by a television opportunities, fairness in economic behavior, and
network. avoidance of concentrated political power resulting
from economic power. Competition results in greater
Answer (C) is incorrect because profits are not output and lower prices than other market structures.
regulated. Radio and TV station owners may make Agreements to limit output of a product would lessen
whatever profits they can as long as the airwaves are competition and increase the price to consumers.
used consistently with "the public interest,
convenience, and necessity."
[11] Source: CMA 1287 1-18
Answer (D) is incorrect because program content is
not censored (but inappropriate programming can Answer (A) is incorrect because professionals and
result in future loss of license). their organizations are not exempt from the antitrust
laws.

[8] Source: CMA 1287 1-14 Answer (B) is incorrect because agreements among
competitors to increase fees are illegal per se.
Answer (A) is incorrect because tariffs are to protect
businesses, not society as a whole. Answer (C) is incorrect because fee discrimination is
not illegal per se since many factors influence the fees
Answer (B) is correct. A social regulation is one for that a particular professional might charge. For
the good of society as a whole. Its purpose is to example, a professional might charge a lower price if
reach behavior that is thought to be not purely a service is to be performed during a normally slow
financial or economic. The prohibition of period.
discrimination in employment and housing are
examples. Answer (D) is correct. Lawyers, public accountants,
and other professionals have received scrutiny from
Answer (C) is incorrect because patent laws are to the FTC in recent years concerning pricing policies.
encourage new product development and protect In general, professionals cannot make agreements
inventors. that provide for either minimum or maximum fees
since price fixing by collusion among competitors is a
Answer (D) is incorrect because such rules regulate per se violation of the Sherman Act.
economic activity.

[12] Source: CMA 1287 1-19


[9] Source: CMA 1287 1-16
Answer (A) is incorrect because the Commerce
Answer (A) is correct. The Clayton Act prohibits Department has no authority to enforce the antitrust
exclusive dealing arrangements in which the effect laws.
may be to substantially lessen competition or tend to
create a monopoly in any line of commerce. Answer (B) is incorrect because the Justice
Department also has the authority to enforce antitrust Answer (B) is incorrect because the FTC does not
laws. regulate import quotas. Its operations are purely
domestic.
Answer (C) is correct. The FTC, in conjunction with
the antitrust division of the Justice Department, has Answer (C) is incorrect because the Interstate
broad authority to enforce the antitrust laws. Since Commerce Commission regulates railroads and the
mergers may lessen competition or tend to create a Federal Aviation Administration regulates airlines.
monopoly under the terms of the Clayton Act, they
are scrutinized by the FTC and the Justice Answer (D) is incorrect because labor unions are
Department. monitored by the Department of Labor.

Answer (D) is incorrect because prohibitions against


mergers have not been repealed. [16] Source: CMA 1289 1-24

Answer (A) is incorrect because it is a provision of


[13] Source: CMA 1287 1-20 the Taft-Hartley Act.

Answer (A) is incorrect because users who receive Answer (B) is correct. The Labor-Management
different volumes of service usually pay different Relations Act of 1947, or Taft-Hartley Act, placed
rates. restraints on unions that resulted in a more even
balance of power between labor and management.
Answer (B) is incorrect because rates are also based The Act provides for mediation of disputes, prohibits
on other than original cost when facilities are acquired closed shops, encourages the individual states to pass
by one utility from another utility. right-to-work laws, defines unfair labor practices,
and provides for an 80-day cooling-off period. It
Answer (C) is incorrect because utilities are does not guarantee workers the right to organize.
monopolies and are not permitted to refuse to deal This right is guaranteed by the Wagner Act of 1935.
with paying customers.
Answer (C) is incorrect because it is a provision of
Answer (D) is correct. Public utilities are natural the Taft-Hartley Act.
monopolies subject to governmental price controls.
State utility commissions establish utility rates by Answer (D) is incorrect because it is a provision of
setting rates so that capital providers earn a the Taft-Hartley Act.
reasonable return on their investment (the rate base).
The rate base is the amount of assets minus
depreciation used in the company's operations. [17] Source: CMA 1289 1-29

Answer (A) is incorrect because federal law applies


[14] Source: CMA 1287 1-21 in all states and supplements state blue-sky
(securities) laws.
Answer (A) is incorrect because an increase in
variable costs would decrease the number of firms in Answer (B) is correct. The basic purpose of the
the industry. federal securities laws is to provide disclosure of
adequate information so that investors can evaluate
Answer (B) is incorrect because fixed costs will be investments. This is accomplished through complex
unaffected. registration and reporting requirements concerning the
issuance and subsequent trading of securities.
Answer (C) is incorrect because prices will increase However, the federal government does not assess the
in the long run if marginal cost increases. merits of these securities.

Answer (D) is correct. An increase in variable costs Answer (C) is incorrect because the SEC does not
will cause an increase in prices in both the short and determine the merits of securities; it evaluates whether
long run since the variable costs will have to be sufficient information is provided.
covered in both periods. If only fixed costs were
increased, only the long run might be affected. Answer (D) is incorrect because the securities laws
are not intended to influence the investment of capital
in more socially or economically beneficial ways.
[15] Source: CMA 1289 1-26

Answer (A) is correct. The Federal Trade [18] Source: CMA 1289 1-30
Commission Act of 1914 created the Federal Trade
Commission (FTC) to help provide day-to-day Answer (A) is correct. An administrative agency is a
enforcement of the antitrust laws. In addition to public board, commission, officer, etc. (other than a
having jurisdiction to enforce all of the antitrust laws, judicial or legislative body), with limited power to
the FTC has broad authority to prevent unfair enforce the law, make rules, and adjudicate disputes
methods of competition and unfair or deceptive acts involving private rights and duties. It may be
or practices. The FTC covers virtually all aspects of independent (the SEC, FTC, FCC) or executive
commerce. Any form of commercial deception is of (OMB, cabinet departments). These agencies affect
concern to the FTC. almost all aspects of the nation's social, economic,
commercial, and political life. The activity of an
administrative agency is an exception to the total amount of payments must be disclosed.
separation of powers doctrine because the agency Open-end credit transactions, such as those involving
has the power to enforce the law (executive power), credit cards, also have specific, detailed disclosure
make rules and regulations (legislative power), and requirements.
decide disputes (judicial power). Because an agency
is not a formally separate branch of government, it
exercises only that power delegated to it, and any act [21] Source: CMA 0690 1-6
outside the scope of the delegation is unconstitutional.
Answer (A) is incorrect because appropriate tests
Answer (B) is incorrect because the GAO is the audit can be given to all applicants.
arm of Congress and has no approval authority.
Answer (B) is incorrect because the tests are not
Answer (C) is incorrect because proposed legal if they are not directly related to job
regulations are made public in the Federal Register. requirements.

Answer (D) is incorrect because there is no 1-year Answer (C) is incorrect because the tests are legal if
notification requirement. related to job requirements. Professionally developed
tests that meet EEOC guidelines are most likely to
survive scrutiny.
[19] Source: CMA 0690 1-2
Answer (D) is correct. The equal employment
Answer (A) is incorrect because the Consumer opportunity laws are designed to prohibit
Product Safety Act of 1972 exempts certain discrimination in employment. Employers do,
products, e.g., aircraft, motor vehicles, food, drugs, however, have the right to use selection procedures in
and cosmetics. hiring as long as the procedures are not
discriminatory. Thus, an employer will have a defense
Answer (B) is incorrect because a ban on a product under Title VII of the Civil Rights Act of 1964 if it
is appropriate only if specific standards will be uses professionally developed pre-employment ability
ineffective. tests that are directly related to job requirements.

Answer (C) is incorrect because industry standards


are encouraged. [22] Source: CMA 0691 1-21

Answer (D) is correct. The CPSC promotes Answer (A) is incorrect because the EEOC deals
voluntary safety standards, develops and enforces mostly with large companies.
mandatory standards, prohibits unsafe products if
safety standards will not be sufficient, recalls Answer (B) is incorrect because the EEOC is not
hazardous products, furnishes information to concerned with employee productivity.

consumers, and works with state and local Answer (C) is incorrect because the EEOC attempts
governments. Thus, if a product is hazardous and to conciliate disagreements on matters within its
presents an unreasonable risk of injury and safety authority, but it has no major policy not to seek legal
standards will not adequately protect the public, the remedies, such as court orders, damage awards, or
CPSC may issue an appropriate order banning the loss of government contracts.
product. If a hazard is deemed to be imminent, it may
seek a U.S. District Court's authorization to seize the Answer (D) is correct. The EEOC has set goals and
product or halt its distribution. timetables for employers to bring female and minority
work forces up to the appropriate percentages as
they relate to the available labor pool. The means of
[20] Source: CMA 0690 1-5 achieving these goals, such as affirmative action
programs, are often highly controversial.
Answer (A) is incorrect because the Consumer

Credit Protection Act merely requires disclosure. It [23] Source: CMA 0691 1-24
does not regulate interest rates.
Answer (A) is incorrect because fair trade laws
Answer (B) is incorrect because the Act did not pertain to the prices charged by retailers.
prohibit the use of appraisal fees.
Answer (B) is incorrect because the term fair
Answer (C) is incorrect because the Act had nothing employment laws is nonsensical in this context.
to do with wage garnishment.
Answer (C) is incorrect because the term open shop
Answer (D) is correct. The Truth-in-Lending Act laws is not customarily used to describe laws that bar
applies to creditors that extend consumer credit to conditioning employment upon union membership.
individual debtors (not organizations) in amounts of
$25,000 or less. For a closed-end credit transaction, Answer (D) is correct. State right-to-work laws are
e.g., the typical car loan, the total finance charge, expressly permitted by the NLRA. They allow
annual percentage interest rate, amount financed, late individuals to work for a unionized employer without
charges, security interest held by the creditor, the being compelled to join the union or pay dues.
number and amounts of payments, due dates, and the Moreover, unions must represent all employees in the
bargaining unit, including these free riders. sliding tax charge based on the amount of effluent
emitted. This method is preferred because, as effluent
discharge increases, a company's tax increases,
[24] Source: CMA 0691 1-30 providing a strong incentive for firms to discover new
methods of controlling pollution. Rather than dictating
Answer (A) is incorrect because the airline industry technology, the tax allows firms to seek out the
has been deregulated. technology that is the most cost effective.

Answer (B) is incorrect because the banking industry Answer (C) is incorrect because the EPA would not
has been deregulated. be directly involved in technology.

Answer (C) is incorrect because the trucking industry Answer (D) is incorrect because the tax would be an
has been deregulated. indirect method of economic intervention.

Answer (D) is correct. In recent years, many


industries that were formerly highly regulated have [27] Source: CMA 1291 1-21
been deregulated to some extent. Airlines, banks,
railroads, and trucking have all been deregulated. The Answer (A) is incorrect because all regulated
steel industry has never been highly regulated. Thus, industries are not exempt from antitrust laws.
there was no opportunity for steel to be deregulated.
Answer (B) is incorrect because some regulated
industries are exempt or partially exempt from the
[25] Source: CMA 1291 1-19 antitrust laws.

Answer (A) is incorrect because the Sherman Act Answer (C) is incorrect because Congress, not the
makes illegal every restraint of trade in interstate or applicable regulatory agency, determines which
foreign commerce. industries are exempt from specific laws.

Answer (B) is incorrect because the Sherman Act Answer (D) is correct. Regulated industries such as
prohibits resale price maintenance. public utilities and banks may not be covered by the
U.S. antitrust laws. For example, the need for
Answer (C) is correct. The various exemptions from monopoly status and tight regulation by state agencies
antitrust laws include intrastate commerce, labor make energy utilities essentially exempt from antitrust
unions, regulated utilities, patents, copyrights, and laws. Banks and insurance companies, however, are
reasonable noncompetition clauses between buyers less tightly regulated and are subject to some
and sellers of businesses, partners in a partnership, provisions of the antitrust laws. Thus, different laws
and purchasers of technology or equipment. apply to different regulated industries.
Agricultural and fishing organizations, financial
institutions, broadcasting, transportation industries,
and professional baseball also have varying levels of [28] Source: CMA 1291 1-22
exemption. Moreover, in 1982, Congress enacted
the Export Trading Company Act. The purpose of Answer (A) is correct. A federal regulatory agency
the ETCA is to encourage export of U.S. produced may regulate some aspect of all industries or may
goods and services. Persons who wish to form an regulate a specific industry in accordance with power
export trading company may obtain from the delegated by Congress in the enabling legislation.
Commerce Department a certificate of antitrust Agency functions include executive, adjudicatory, and
immunity (called a certificate of review) after rulemaking activities. Such agencies, however, may
concurrence by the Justice Department. This not impose taxes.
certificate is issued if certain conditions are met.
These prohibit unfair export competition, substantial Answer (B) is incorrect because federal regulatory
lessening of competition or restraint of trade in the agencies have the power to issue rules and
U.S., unreasonable effects on U.S. prices, and sale regulations.
or resale of the goods or services in the U.S.
Answer (C) is incorrect because federal regulatory
Answer (D) is incorrect because the Sherman Act agencies have the power to investigate violations of
makes illegal every restraint of trade in interstate or statutes and rules.
foreign commerce.
Answer (D) is incorrect because federal regulatory
agencies have the power to recommend penalties for
[26] Source: CMA 1291 1-20 violations of statutes and rules.

Answer (A) is incorrect because pollution would not


be totally eliminated. Some firms might find that a low [29] Source: CMA 1291 1-24
level of pollution is more cost beneficial than total

elimination. Answer (A) is correct. Title VII of the Civil Rights


Act of 1964 prohibits employment discrimination on
Answer (B) is correct. The setting of effluent the basis of race, color, national origin, religion, or
standards has often been criticized as an inefficient sex. Under the Act, a pervasive pattern or practice of
method of pollution control. Economists prefer a discrimination, the employer's adoption of a neutral
rule having an adverse impact on a protected class,
and the adoption of a neutral rule that perpetuates Answer (B) is correct. Economic regulation usually
past discrimination are violations that often must be concerns price and service to the public and is
proved, at least in part, by statistical evidence. A ordinarily industry specific. Social regulation has
controversial remedy sometimes adopted in such broader objectives and more pervasive effects. It
cases is an affirmative action order, which provides addresses quality of life issues, such as workplace
preferences to members of the class that previously and product safety, pollution, and fair employment
suffered from discrimination. Affirmative action practices, and it applies to most industries. Social
preferences apply even though the specific persons regulation has been criticized on the grounds that it is
benefited are not necessarily those who were costly, contributes to overregulation, may inhibit
victimized by illegal discrimination. Affirmative action innovation, increases inflation and may place a
programs are sometimes criticized because disproportionate burden on small companies, thereby
employment preferences are often viewed as reverse having an anticompetitive effect. Another criticism is
discrimination. Moreover, they may not result in the that regulators are perceived to have little concern for
hiring, retention, and promotion of the most the relation of marginal benefits and costs. For
productive workers. example, achieving total compliance with the
standards for treated wastewater may be twice as
Answer (B) is incorrect because a work force in the costly as attaining 95% of those standards. In other
same proportion as the demographics of the total words, the relatively small improvements sometimes
population is not in conflict with the equal sought by regulators may be viewed as excessively
employment opportunity laws. costly by those who are regulated.

Answer (C) is incorrect because even a bona fide Answer (C) is incorrect because OSHA and the
seniority system may violate Title VII if it is found to EPA are criticized for maintaining rigid standards that
perpetuate past discrimination. An affirmative action are mandated regardless of cost.
remedy would necessarily have the effect of
weakening such a system. Answer (D) is incorrect because compliance costs
are often quite high.
Answer (D) is incorrect because wages should be
directly related to marginal productivity.
[32] Source: CMA 1291 1-28

[30] Source: CMA 1291 1-25 Answer (A) is incorrect because the Act does not
require financial reports by management.
Answer (A) is incorrect because the CPSC does not
levy fines, although it may bring suit for violations of Answer (B) is incorrect because the Taft-Hartley Act
the Act, and a court may levy fines as a result. of 1947 designates a list of unfair labor practices on
the part of unions.
Answer (B) is correct. The Consumer Product Safety
Act created the Consumer Product Safety Answer (C) is correct. The National Labor Relations
Commission, which promotes voluntary safety Act, or Wagner Act, was designed to control the
standards, develops and enforces mandatory supply of labor while guaranteeing workers the right
standards, prohibits unsafe products if safety to organize, that is, to form, join, and assist labor
standards will not be sufficient, recalls hazardous unions. It also prohibited certain unfair labor practices
products, furnishes information to consumers, and by management. Moreover, it permits employees to
works with state and local governments. Thus, if a elect a collective bargaining agent by majority vote
product is hazardous and presents an unreasonable and provides for the NLRB. The NLRB is a federal
risk of injury and safety standards will not adequately agency that hears representation cases and
protect the public, the CPSC may issue an determines whether actions by labor or management
appropriate order banning the product. If a hazard is constitute unfair labor practices.
deemed to be imminent, it may seek a U.S. District
Court's authorization to seize the product or halt its Answer (D) is incorrect because the Landrum-Griffin
distribution. However, the emphasis of the Act of 1959 democratized unions.
Commission's work is the prevention of problems
through standard setting, not the punishment of
wrongdoers. [33] Source: CMA 1291 1-29

Answer (C) is incorrect because consumer choice is Answer (A) is incorrect because the Act did not
lessened when the CPSC keeps unsafe products off permit strikes for health and safety violations.
the market.
Answer (B) is incorrect because the union shop is still
Answer (D) is incorrect because the cost of allowed in many states; the closed shop was
regulation results in higher costs for regulated outlawed.
products.
Answer (C) is incorrect because the Act limited the
power of unions, not of employers.
[31] Source: CMA 1291 1-26
Answer (D) is correct. After the NLRA was passed
Answer (A) is incorrect because quality of life is the in 1935, unions became very powerful, strikes were
primary objective of these laws. crippling, and employees were coerced by labor
bosses. The Taft-Hartley Act placed restraints on adequate information. The Securities Acts of 1933
unions that resulted in a more even balance of power and 1934 were not a direct result of the stock market
between unions and employers. Among its many crash; instead, it was the bankruptcy of Ivar Kreuger,
provisions, the 1947 Act prohibited certain unfair the greatest swindler the world has ever known, that
labor practices by unions: coercion of employees to led to the passage of the securities acts.
join unions, discrimination against nonunion
employees except when a valid union shop agreement
is in place, refusing to bargain in good faith, [36] Source: CMA 0693 1-14
secondary strikes, featherbedding (payment by
employers for work not performed), and charging Answer (A) is incorrect because the purpose of a
new members excessive initiation fees. private placement is to avoid SEC regulation.

Answer (B) is incorrect because, in the absence of


[34] Source: CMA 1291 1-30 federal regulation, private placements typically take
less time than public offerings.
Answer (A) is incorrect because comparable worth
does not address comparability of U.S. and foreign Answer (C) is correct. Private placements in
wages. unlimited amounts are exempted from the Securities
Act of 1933. A private placement may be offered
Answer (B) is correct. The equal employment only to knowledgeable and sophisticated investors,
opportunity laws require employees to receive equal may be offered to no more than 35 nonaccredited
compensation for equal work. But compensation purchasers, and may not be offered to the general
based on comparable worth would require workers public. However, the securities are restricted, and the
to be paid for the value of their contributions to the issuer must exercise reasonable care in determining
organization or to society, not on the basis of the that purchasers are not underwriters and are
value established by market forces that may reflect purchasing solely for their own investment purposes.
entrenched sexist attitudes. Thus, workers who Private placement flotation expenses may be lower
perform comparable but not equal work, that is, than with a public offering, but some flotation costs
different jobs needing different skills, would be will still be incurred.
compensated equally. Comparable worth advocates
believe that the application of this principle would Answer (D) is incorrect because a private placement
remedy the underpayment of persons in occupations of securities can be tailor-made to the borrower's
historically dominated by women. The most severe needs.
application of comparable worth law exists in the
Canadian Province of Ontario where it has invoked
much criticism because of the high costs relative to [37] Source: CMA 0693 1-13
the benefits derived. Some Ontario companies have
spent many thousands of dollars determining the Answer (A) is incorrect because blue-sky laws are
extent of a comparable w orth problem only to have state, not federal, financial regulatory laws.
the result show that one or two female workers were
underpaid by a few hundred dollars. Answer (B) is incorrect because blue-sky laws are
state laws and have nothing to do with goodwill.
Answer (C) is incorrect because comparable worth
does not address wage rates for the same job in Answer (C) is correct. Blue-sky laws are state laws
union and non-union environments. designed to prevent fraudulent or misleading security
issues. The name comes from some of the earliest
Answer (D) is incorrect because comparable worth laws that prohibited "everything under the blue skies
does not address wage rates for the same job in the that is fraudulent."
government and the private sector.
Answer (D) is incorrect because blue-sky laws apply
to all companies in a given state.
[35] Source: CMA 0693 1-11

Answer (A) is correct. The 1934 Act was designed [38] Source: CMA 0693 1-19
to regulate securities after initial issuances. Other
purposes of the 1934 Act were to provide adequate Answer (A) is incorrect because small businesses do
information to investors and to prevent insiders from have a significant impact on jobs and the
unfairly using their information. environment; however, they often cannot absorb the
additional costs brought on by regulation.
Answer (B) is incorrect because the Securities Act of
1933 was designed to regulate new issues of Answer (B) is incorrect because Congress has
securities. elected to exempt small businesses from some social
regulation because of the cost burden on such firms.
Answer (C) is incorrect because the objective is not
to protect investors from losses, but to provide Answer (C) is correct. Social regulation embraces a
investors with adequate information upon which to variety of government requirements related to matters
base investment decisions. such as consumer protection, environmental quality,
employment discrimination, and workplace safety.
Answer (D) is incorrect because the objective was to Small businesses have sometimes been exempted
regulate securities and provide investors with from social regulations, because Congress has felt
that compliance costs would be an excessive burden behavior or the nature of its advertising. Boycotts
for small firms. These costs are relatively fixed and have sometimes been quite successful. Shareholder
are therefore higher per unit of output for smaller than resolutions have also been made at stockholder
for larger firms. Compliance costs therefore create a meetings, but such resolutions have rarely been
competitive disadvantage for small firms relative to successful. Lobbying for new laws is also an often
large firms. successful tactic. Occasionally, cooperating with the
manufacturer to find common ground has been used.
Answer (D) is incorrect because the cost burden is a Creating a competing company, however, is not a
more critical factor than small business lobbyists. consumerism tactic -- at least not yet.

[39] Source: CMA 0693 1-20 Answer (D) is incorrect because cooperative
meetings are sometimes used by consumer groups.
Answer (A) is incorrect because the standards were
established without consideration for costs of
compliance. [42] Source: CMA 0693 1-23

Answer (B) is correct. Under the Clean Air Act, Answer (A) is incorrect because OSHA encourages
individuals can initiate or participate in civil labor-management committees to formulate safety
enforcement actions. Individuals may also seek and health programs.
damages for personal injuries suffered as a result of
violations. Answer (B) is incorrect because inspections are
intended to determine whether OSHA standards are
Answer (C) is incorrect because national ambient air being followed. These standards relate to workplace
quality standards (NAAQS) have been established to safety and health.
prevent damage to items such as visibility, crops,
waterways, and buildings. Answer (C) is incorrect because OSHA has the
authority to levy civil monetary penalties of up to
Answer (D) is incorrect because national ambient air $1,000 per violation and up to $10,000 for a repeat
quality standards (NAAQS) have been established offense.
with respect to specified pollutants to protect human
health and the public welfare. Answer (D) is correct. OSHA is empowered to
conduct surprise inspections to determine whether
standards are being met. However, an employer may
[40] Source: CMA 0693 1-21 demand that the inspector obtain a search warrant.
Thus, no precipitating event must occur prior to an
Answer (A) is incorrect because the CPSC is not inspection.
dependent upon manufacturers to provide information
on product safety. Much information is obtained from
consumers. [43] Source: CMA 0693 1-24

Answer (B) is incorrect because standards are Answer (A) is incorrect because most regulated
compulsory. industries, such as public utilities, airlines, and banks,
are very capital intensive.
Answer (C) is incorrect because cost-benefit analysis
is not a consideration when products are dangerous. Answer (B) is incorrect because uncontrolled
monopoly power may result in excessive prices, poor
Answer (D) is correct. A manufacturer, retailer, or quality, and inadequate output.
distributor must notify the CPSC if it has reason to
know that a product poses a "substantial product Answer (C) is incorrect because regulated industries
hazard" because it violates a CPSC safety rule or for are often considered to be vital. Indeed, some are
other reasons. In these circumstances, the CPSC deemed to be sufficiently vital to justify public
may order the party to notify those affected, repair or ownership or the equivalent (e.g., the postal service).
replace the product, or submit its own corrective
action plan. Answer (D) is correct. Regulated industries are
usually those in which natural monopolies exist.
Regulation is imposed to prevent the abuses that may
[41] Source: CMA 0693 1-22 flourish in a noncompetitive environment and to
ensure that consumers receive adequate service at
Answer (A) is incorrect because boycotts are often fair prices. Rates are established to provide a fair and
used by consumer groups attempting to influence a reasonable return to investors, not consumers.
manufacturer's behavior.

Answer (B) is incorrect because shareholder [44] Source: CMA 0693 1-26
resolutions are sometimes used in an attempt to
influence the boards of large corporations. Answer (A) is incorrect because the Justice
Department and the FTC can enforce antitrust laws.
Answer (C) is correct. Boycotts of a manufacturer's
products are the most common tactic used by Answer (B) is incorrect because the a state attorney
consumer groups to induce a company to change its general and the FTC can enforce antitrust laws.
Answer (C) is incorrect because individual citizens Answer (A) is incorrect because the ADA bans
and the FTC can enforce antitrust laws. employment discrimination against people with
disabilities.
Answer (D) is correct. Under the Sherman Act,
violations may be civil or criminal. Criminal penalties Answer (B) is incorrect because the ADA provides
include fines and imprisonment and enforcement is by tax incentives for compliance costs.
the Justice Department. Civil suits may also be
brought by private parties, who may recover treble Answer (C) is incorrect because the ADA bans
damages. The FTC also has specific authority to discrimination against people with disabilities in
enforce the Clayton and Robinson-Patman Acts and transportation.
implied authority to enforce the Sherman Act. State
law may also provide remedies for antitrust violations. Answer (D) is correct. Title I prohibits employment
The U.S. Department of Commerce is not entitled to discrimination against qualified individuals with
bring suit for violation of antitrust laws. disabilities with respect to job applications, hiring,
promotion, training, pay, and termination. The Act
also establishes requirements for employers and
[45] Source: CMA 0693 1-25 providers of public accommodations, public
transportation, and telecommunications. Employers
Answer (A) is correct. The Sherman Act specifies must make "reasonable accommodations" for
that some type of arrangements between competitors individuals with disabilities that do not impose an
are to be considered unreasonable without inquiry. undue burden, for example, by modifying facilities
These are called per se violations. These include: and work schedules, obtaining equipment, etc.
price fixing (agreeing to any price), division of However, no federal funds are provided for
markets (agreeing where to sell), group boycotts implementation of ADA by employers.
(agreeing not to deal with another), and resale price
maintenance (limitations on buyer's resale price). The
most critical of these in terms of number of violations [48] Source: CMA 0693 1-29
prosecuted and dollars involved is price fixing.
Answer (A) is incorrect because quality standards
Answer (B) is incorrect because allocation of were established by the Environmental Protection
markets, although a per se violation, has rarely been a Agency.
problem.
Answer (B) is incorrect because fees have been
Answer (C) is incorrect because boycotts involving established for firms that emit pollutants.
third parties are not nearly as common as price fixing
prosecutions. Answer (C) is correct. Environmental laws have
established quality standards with respect to water,
Answer (D) is incorrect because tie-in sales are land, and air. One approach has permitted violations,
prohibited by the Clayton Act of 1914. but subjects violators to sliding charges depending
upon the quantity of pollutants released. Companies
that are below the standard for a particular pollutant
[46] Source: CMA 0693 1-27 can sell their rights to other firms in the industry. In
addition, tax laws have provided incentives for
Answer (A) is incorrect because many cases have environmental improvements, for example a tax credit
been filed under the Sherman Act; however, the rule for individuals who installed insulation. Similarly,
of reason has limited the number of successful windmills are encouraged by the tax laws. Moving
prosecutions. facilities to offshore sites where governmental
standards are less stringent might be undertaken by
Answer (B) is incorrect because numerous individual companies, but such action is not a
companies still engage in anti-competitive conduct. governmental device.

Answer (C) is incorrect because intent is significant; Answer (D) is incorrect because companies may buy
for example, a company that has a monopoly thrust and sell their rights to emit pollutants.
upon it is not in violation.

Answer (D) is correct. The Sherman Antitrust Act of [49] Source: CMA 0693 1-30
1890 makes illegal every contract, combination, or
conspiracy in restraint of trade in interstate or foreign Answer (A) is incorrect because the FDA can extend
commerce. Successful prosecutions have been few patent lives to allow for time lost during the regulatory
because the courts developed a rule of reason review.
stipulating that only unreasonable restraints of trade
are illegal. Thus, courts balance the anti-competitive Answer (B) is incorrect because the FDA considers
effects against the pro-competitive effects of a benefit-risk tradeoffs.
restraint of trade. Unless a restraint is unreasonable
by its very nature (a per se violation), the rule of Answer (C) is incorrect because the FDA maintains
reason applies. purity standards.

Answer (D) is correct. The FDA was created by the


[47] Source: CMA 0693 1-28 Federal Food, Drug, and Cosmetics Act of 1938 to
help maintain the safety of drugs, food, cosmetics, safety, environmental degradation, and fair
and medicinal products and devices. It regulates the employment practices. The abuses addressed are
testing, distribution, and sale of drugs. Upon receiving those that are difficult for market forces to remedy.
a new drug application, the FDA conducts hearings For example, consumers may purchase products on
and investigates the merits of the drug in a process the basis of price and quality, but without regard to
that may require years. Thus, an application must be the environmental impact of their production, and
supported by substantial evidence that the drug will unsafe working conditions may be tolerated by
have the asserted effects. individuals who have few opportunities for other
employment.

[50] Source: CMA 1293 1-1 Answer (B) is incorrect because, although consumer
and environmental groups may occasionally exercise
Answer (A) is incorrect because the Sherman Act some lobbying power, they are typically underfunded
also applies to domestic corporations. and would have little impact on legislatures in the
absence of an obvious need for social regulation.
Answer (B) is incorrect because the Taft-Hartley Act
of 1947 outlawed closed shops. Answer (C) is incorrect because there is great
difficulty in measuring both the benefits and costs of
Answer (C) is incorrect because monopolies in most social regulation.
interstate or foreign commerce are illegal; monopolies
in intrastate commerce are not covered by the Answer (D) is incorrect because no regulation is
Sherman act. Also, a company must show an desirable, but some regulation is necessary when
objective interest to monopolize. Monopoly power is market forces are ineffective.
lawful if it is obtained by superior businesses acumen.

Answer (D) is correct. Section 1 of the Sherman [53] Source: CMA 1293 1-5
Antitrust Act of 1890 makes illegal every contract,
combination, or conspiracy in restraint of trade in Answer (A) is incorrect because the Clean Air Act
interstate or foreign commerce. Some types of does not make pollution illegal; it provides
arrangements between competitors are found comprehensive regulation of air quality.
unreasonable without inquiry. These are per se
violations and include price fixing, division of markets, Answer (B) is incorrect because regulating the
group boycotts, and resale price maintenance. amount of pollution a specific company can emit is
Section 2 prohibits the acts of monopolizing or not efficient. That is why the emphasis is on the
attempting to monopolize. application of air quality standards in the air quality
control regions in the states. Moreover, the states are
required to identify the major, not all, sources of
[51] Source: CMA 1293 1-2 pollution.

Answer (A) is incorrect because the forces of supply Answer (C) is incorrect because the Act does not
and demand do not determine prices; a regulatory promote social awareness about the harm due to
agency sets limits on the price a natural monopoly can pollution; in fact, it allows some pollution if a
charge. company is willing to pay to pollute.

Answer (B) is incorrect because firms cannot easily Answer (D) is correct. The Clean Air Act allows
enter the utility industry. Great amounts of capital are companies to pollute legally if they have a permit to
needed to establish a natural monopoly. do so. Firms can buy or sell these permits, and the
result is essentially a free market that enables a firm to
seek out a technology that is most cost effective. If
Answer (C) is incorrect because quality standards the price of permits is too high, a firm will buy new
are set by the same regulatory agency that controls equipment that will not pollute. If permits can be
prices. acquired at a low cost, a company may decide to
continue producing with its old, high-pollution
Answer (D) is correct. Regulated industries such as equipment.
gas and electric utilities, most of which are natural
monopolies, have price and quality controls
established by state or local governmental regulatory [54] Source: CMA 1293 1-6
bodies. Such industries usually have inelastic demand
curves, which means regulation can help control Answer (A) is incorrect because a merger between
prices. Public utilities are monopolies because having firms in different and unrelated markets is a
more than one of each type of utility in a locality conglomerate merger.
would be inefficient. Rates are normally set to
provide owners with a reasonable return on their Answer (B) is incorrect because a merger between
investment. two or more firms at different stages of the
production process is a vertical merger.

[52] Source: CMA 1293 1-4 Answer (C) is incorrect because a merger between a
producer and a supplier is a vertical merger.
Answer (A) is correct. Social regulation concerns
quality of life issues, e.g., workplace and product Answer (D) is correct. A horizontal merger is one
between competitors in the same market. From the management without submitting to the complexities of
viewpoint of the Justice Department, it is the most corporate formation and operation.
closely scrutinized type of merger because it has the
greatest tendency to reduce competition. Answer (C) is correct. The most common form of
business organization is the sole proprietorship. A
proprietorship is easily and inexpensively organized
[55] Source: CMA 1293 1-7 because no formalities are required. Moreover, it
consists of one person (the proprietor). It is created
Answer (A) is incorrect because unions raise wages under common law whenever a person enters into
for their members and indirectly reduce wages in the business and does not choose another form or
nonunionized labor market. organization.

Answer (B) is correct. Labor unions have typically Answer (D) is incorrect because a corporation is a
worked for higher wages and better working legal entity with a legal existence separate from its
conditions for their members. Through collective owners; the formation and existence of corporations
bargaining and restrictions on membership, unions is entirely regulated by state statute.
increase the wage rates of members in comparison
with those earned by nonunionized workers. Because
of the greater cost of hiring unionized workers, the [58] Source: CMA 1293 1-11
amount of labor demanded by employers is reduced.
The workers that are unemployed as a result may Answer (A) is incorrect because the elimination of all
then seek employment in the nonunion labor market, pollution would probably be too costly for the
thereby increasing the supply and decreasing the resulting benefits.
wages in that market.
Answer (B) is incorrect because firms are more
Answer (C) is incorrect because unions raise wages supportive of cost-benefit regulatory policies.
for their members and indirectly reduce wages in the
nonunionized labor market. Answer (C) is incorrect because granting tax relief to
those firms which pollute the least is not a
Answer (D) is incorrect because unions raise wages cost-benefit policy.
for their members and indirectly reduce wages in the
nonunionized labor market. Answer (D) is correct. If environmental policies
weigh the costs and benefits of regulation, some
pollution will exist because the costs of eliminating all
[56] Source: CMA 1293 1-8 pollution will likely outweigh the benefits.
Cost-benefit considerations are reflected in, for
Answer (A) is correct. A closed shop is a workplace example, the establishment of air quality standards
in which union membership is required as a condition that do not attempt to eliminate the incidence of
of obtaining employment. The closed shop was specified pollutants and requirements for control
outlawed by the Labor-Management Relations Act technology that, for the most part, do not mandate
(Taft-Hartley Act) of 1947. In contrast with a closed use of devices that achieve the maximum pollution
shop, a union shop is still allowed in some states. In a reduction. The drawback of a cost-benefit approach
union shop, an employee is required to join the union is the difficulty of quantifying benefits.
after employment. Still other states have
right-to-work laws that allow employees to work at
any job without union membership. [59] Source: CMA 1293 1-12

Answer (B) is incorrect because, in a closed shop, all Answer (A) is incorrect because the Federal Reserve
employees must be a member of the union. System regulates banks, but does not insure deposits.

Answer (C) is incorrect because collective bargaining Answer (B) is incorrect because the FSLIC was the
is necessary in a closed shop. All workers are federal agency that formerly insured savings and loan
members of the union. associations; the FSLIC went bankrupt as a result of
the savings and loan crisis of the 1980s.
Answer (D) is incorrect because only workers who
are union members have the right to work in a closed Answer (C) is correct. Commercial bank deposits
shop. are insured up to $100,000 per account by the
Federal Deposit Insurance Corporation (FDIC), a
federal agency. It is funded by premiums paid by
[57] Source: CMA 1293 1-9 member banks.

Answer (A) is incorrect because a partnership is Answer (D) is incorrect because the treasury is not
more complex than a proprietorship, but less difficult the primary insurer of bank deposits, although it may
to form than a corporation. ultimately be asked by Congress to provide funds if
the FDIC encounters financial difficulties.
Answer (B) is incorrect because a limited partnership
can be created only pursuant to a statute. It is a form
of business organization unknown at common law. A [60] Source: Publisher
limited partnership permits investors to avoid
personal liability and the duties of day-to-day Answer (A) is incorrect because, under NEPA,
federal agencies must give environmental
considerations a weight equal to but not greater than Answer (A) is incorrect because profit percentages
that afforded nonenvironmental concerns. NEPA are not set by antitrust laws other than to the extent
augments the existing powers of federal agencies to that price discrimination is prohibited.
deal with these environmental matters.
Answer (B) is incorrect because firms may enter into
Answer (B) is correct. The provisions of NEPA joint ventures.
focus on federal governmental actions. Federal
agencies are specifically directed to incorporate an Answer (C) is correct. Antitrust laws are designed to
analysis of environment consequences in their promote more efficient allocation of resources,
decision-making processes. Actions of private greater choice for consumers, greater business
persons are affected by NEPA only when federal opportunities, fairness in economic behavior, and
involvement (approval, funding, etc.) is necessary avoidance of concentrated political power resulting
before such persons may act (e.g., federal approval from economic power. Competition results in greater
before drilling for oil in ocean waters within U.S. output and lower prices than other market structures.
jurisdiction). Otherwise, NEPA does not directly
concern activities of private persons. Answer (D) is incorrect because competition results
in greater output and lower prices than other market
Answer (C) is incorrect because, under NEPA, structures.
federal agencies must give environmental
considerations a weight equal to but not greater than
that afforded nonenvironmental concerns. NEPA [63] Source: Publisher
augments the existing powers of federal agencies to
deal with these environmental matters. Answer (A) is incorrect because a pricing agreement
among competitors, even if aimed at lowering prices,
Answer (D) is incorrect because, under NEPA, is illegal per se because it is assumed that such an
federal agencies must give environmental agreement over the long run will substantially lessen
considerations a weight equal to but not greater than competition.
that afforded nonenvironmental concerns. NEPA
augments the existing powers of federal agencies to Answer (B) is incorrect because pricing agreements
deal with these environmental matters. aimed at eliminating unfair competition and fixing
prices are illegal.

[61] Source: Publisher Answer (C) is incorrect because it is presumed that in


the long run even reasonable and fair agreements will
Answer (A) is incorrect because the CWA broadly contribute to a lessening of competition.
prohibits any discharges of pollutants into waters
subject to the jurisdiction of the United States by any Answer (D) is correct. No pricing agreements among
person, except in compliance with the Act. Under the competitors are legal because the antitrust laws forbid
CWA, impairment of navigation is irrelevant. competitors to agree on the price of the products
they sell.
Answer (B) is incorrect because to be subject to
federal jurisdiction, and thus the CWA, the waters in
which pollutants are discharged must be so-called [64] Source: Publisher
"navigable waters," which are defined as waters of
the United States (including the territorial waters). Answer (A) is incorrect because many cases have
This broad definition does not encompass all bodies held that 60% or less of the market constitutes
of water located within the bounds of the United monopolistic power. Hence, the firm need not hold
States. 100% of the market.

Answer (C) is incorrect because the Rivers and Answer (B) is incorrect because the government
Harbors Act of the late 1800s was the first major need not prove that the defendant controls a
piece of federal legislation promulgated to protect particular percentage of the market to establish that
U.S. waterways. Until the passage of the CWA, the an illegal monopoly or attempt to monopolize has
Rivers and Harbors Act was also used to combat occurred.
pollutive discharges, although its original purpose was
to keep waterways clear from obstruct ions to Answer (C) is correct. The purpose of antitrust laws
navigation. is to preserve and promote competition. The working
definition of monopoly in antitrust law is that the
Answer (D) is correct. The CWA (1972) defendant has the power to control prices or exclude
substantially amended the Federal Water Pollution competition. Under the Sherman Act, formation of or
Control Act of 1948. It seeks to restore and maintain the attempt to form a monopoly is illegal.
the physical and biological integrity of the waters of
the United States. Its objectives are to render water Answer (D) is incorrect because the Sherman Act
suitable for recreation and propagation of fish and prohibits monopolies and attempts to monopolize.
other wildlife and to eliminate discharges of Thus, proof of contracts, combinations, and
pollutants. conspiracies in restraint of trade is not always
necessary.

[62] Source: Publisher


[65] Source: Publisher Violations of the FCPA are federal felonies. The
penalties are 5 years in prison or a $10,000 fine or
Answer (A) is incorrect because the accounting both for an officer, director, or shareholder who
requirements apply only to publicly held, registered helps make the bribe.
companies under the 1934 act.

Answer (B) is incorrect because the accounting [68] Source: CIA 0594 IV-67
requirements apply only to publicly held, registered
companies under the 1934 act. Answer (A) is incorrect because the easy and
inexpensive organization is one of the most appealing
Answer (C) is correct. The accounting requirements characteristics of sole proprietorships.
of the FCPA apply to all companies required to
register and report under the Securities Exchange Act Answer (B) is incorrect because sole proprietors
of 1934. These companies must maintain books, make all management decisions, and have the rights
records, and accounts in reasonable detail that to all profits as well as the right to sell the business.
accurately and fairly reflect transactions. The FCPA Thus, the entrepreneur has freedom of action.
also requires these companies to maintain a system of
internal accounting control that provides certain Answer (C) is incorrect because, as the sole owner
reasonable assurances, including that corporate and profit-sharer, a sole proprietor has a strong
assets are not used for bribes. If payoffs are made, incentive to manage the business efficiently so as to
they must be reflected in the company's records. earn more profits.

Answer (D) is incorrect because the accounting Answer (D) is correct. The sole proprietor does not
requirements apply only to publicly held, registered enjoy the benefits of specialization. (S)he must
companies under the 1934 act. perform all management functions, including all
decisions relating to buying and selling, the acquisition
and maintenance of personnel, and the technical
[66] Source: Publisher aspects of production, advertising, and distribution.

Answer (A) is correct. The main purpose of the


Foreign Corrupt Practices Act of 1977 is to prevent [69] Source: CIA 0594 IV-66

bribery by firms that do business in foreign countries. Answer (A) is incorrect because a sole
A major ramification is that it requires all companies proprietorship is owned by one person. The sole
that must register with the SEC under the Securities proprietor is personally liable for all debts.
Exchange Act of 1934 to maintain adequate
accounting records and a system of internal Answer (B) is incorrect because general partners
accounting control. share profits and losses of the venture. Debts of a
partnership are ultimately the debts of the individual
Answer (B) is incorrect because, although some general partners.
international accounting standards have been
promulgated, they are incomplete and have not Answer (C) is correct. A shareholder owns a
gained widespread acceptance. property interest in the underlying net assets of the
corporation and is entitled to share in its profits. But,
Answer (C) is incorrect because there are no unlike a sole proprietor or a general partner, the
requirements for providing periodic reports on foreign shareholder is not subject to liability beyond his/her
commerce or foreign political party affiliations. investment.

Answer (D) is incorrect because U.S. auditing Answer (D) is incorrect because monopoly is not a
standards apply to the conduct of audits by legal form of business organization.
accountants.

[70] Source: CIA 0589 IV-47


[67] Source: Publisher
Answer (A) is correct. Unlike a corporation, a
Answer (A) is incorrect because this is not covered partnership is not treated as a separate legal entity for
by the provisions in the FCPA. purposes of liability. A corporate shareholder is
ordinarily liable only to the extent of the investment,
Answer (B) is incorrect because this is not covered but a general partner usually has full personal liability
by the provisions in the FCPA. for the obligations of the partnership.

Answer (C) is incorrect because all U.S. firms are Answer (B) is incorrect because double taxation is a
subject to the anti-bribery provisions. disadvantage of the corporate but not the partnership
form of business organization. Corporate income is
Answer (D) is correct. The Foreign Corrupt taxed at the corporate level, and the individual
Practices Act (FCPA) prohibits any U.S. firm from shareholders pay taxes on dividends. A partnership,
making bribes to foreign officials to influence official however, is not a taxable entity.
acts. The businesses subject to the FCPA include
corporations, partnerships, limited partnerships, Answer (C) is incorrect because partnerships can be
business trusts, and unincorporated organizations. created without formalities if two or more persons
agree to become associated as co-owners to carry the balance of power between labor and
on a business for profit. The agreement need not even management. It outlawed the closed shop, which
be explicit. A corporation, however, must be formed required union membership as a condition of
according to specific statutory requirements. obtaining employment. In addition, the Taft-Hartley
Act required unions to bargain in good faith. Also, the
Answer (D) is incorrect because partnerships are not President was given authority to obtain an injunction
subject to as many regulatory requirements as against strikes for an 80-day cooling-off period if
corporations. national health, welfare, or security was threatened.

Answer (D) is incorrect because the Taft-Hartley Act


[71] Source: CIA 0593 IV-47 did not apply to federal employees.

Answer (A) is incorrect because a limited partnership


is not used to limit withdrawals. [74] Source: CMA 0694 1-18

Answer (B) is incorrect because a limited partnership Answer (A) is correct. The Robinson-Patman Act
is not used to limit the duration of the partnership. prohibits price discrimination with respect to both
buyers and sellers. Price discrimination is allowed if it
Answer (C) is correct. A primary characteristic of is to meet a competitor's price quote or if justified by
limited partnerships is that they must have both the quantity purchased.
general and limited partners. General partners have
unlimited liability for the obligations of the partnership Answer (B) is incorrect because cost increases
and are entitled to manage the business. A limited caused by government specification is not an
partner's liability is ordinarily limited to the amount of
the capital contribution to the partnership. (S)he does allowable exception to the Robinson-Patman Act.
not have a right to participate in management. A
limited partner who nevertheless has a role in Answer (C) is incorrect because the nature of a
management may become personally liable for market is not a factor in price discrimination under the
partnership obligations to third parties aware of that Robinson-Patman Act.
role.
Answer (D) is incorrect because the Justice
Answer (D) is incorrect because a limited partnership Department does not give approval for price
is not used to the authority of general partners to discrimination.
manage the business.

[75] Source: CMA 1294 1-10


[72] Source: CMA 0694 1-16
Answer (A) is incorrect because the Sherman Act of
Answer (A) is incorrect because the Clayton Act of 1890 did not mention tying sales or interlocking
1914 makes price discrimination, tying contracts, directorates.
anti-competitive mergers, and interlocking
directorates illegal. Answer (B) is correct. The Clayton Act of 1914
prohibits mergers or acquisitions of stock that may
Answer (B) is incorrect because unfair competition lessen competition or tend to create a monopoly.
and deceptive business practices are prohibited by Mergers can be prevented by the Justice Department
the Federal Trade Commission Act of 1914. before they occur. Also prohibited by the Clayton
Act are sales that prevent the buyer from dealing with
Answer (C) is incorrect because the Clayton Act of the seller's competitors, tying (or tie-in) sales,
1914 allows the Justice Department to prevent exclusive dealing, price discrimination among different
mergers before they occur. buyers, and interlocking directorates.

Answer (D) is correct. The Sherman Act makes Answer (C) is incorrect because the Antitrust
Improvements Act of 1976 amended the antitrust
illegal every contract, combination, or conspiracy in laws in various ways.
restraint of trade in interstate or foreign commerce.
Section 2 of the act prohibits monopolization or the Answer (D) is incorrect because the Federal Trade
attempt to monopolize. Commission Act of 1914 created the Federal Trade
Commission to enforce the Sherman and Clayton
acts.
[73] Source: CMA 0694 1-17

Answer (A) is incorrect because the closed shop was [76] Source: CMA 1294 1-11
outlawed by the Taft-Hartley Act.
Answer (A) is incorrect because the Taft-Hartley Act
Answer (B) is incorrect because the Taft-Hartley Act provides for an 80-day injunction against a work
did not require binding arbitration. stoppage that might imperil national security.

Answer (C) is correct. The Labor-Management Answer (B) is correct. The Wagner Act gave
Relations Act of 1947, better known as the employees the right to join unions, bargain collectively
Taft-Hartley Act, placed restraints on unions to even with employers, and engage in concerted activities for
collective bargaining. In effect, labor unions were Answer (A) is incorrect because social legislation is
exempted from antitrust laws. The National Labor criticized when standards are rigid.
Relations Board (NLRB) was formed to administer
the Act. Answer (B) is incorrect because compliance costs
are typically high.
Answer (C) is incorrect because the Taft-Hartley Act
of 1947 provided for restraints on unions to permit a Answer (C) is incorrect because the free market is
more even balance of power between labor and normally not relied upon; with certain exceptions,
management. compliance is mandatory.

Answer (D) is incorrect because the Landrum-Griffin Answer (D) is correct. Economic regulation usually
Act of 1959 requires that union officials be elected on concerns price and service to the public and is
a regular basis and that secret ballots be used. It also ordinarily industry specific. Social regulation has
requires that unions submit financial reports to the broader objectives and more pervasive effects. It
government. addresses quality of life issues, such as workplace
and product safety, pollution, and fair employment
practices, and it applies to most industries. Social
[77] Source: CMA 1294 1-12 regulation has been criticized on the grounds that it is
costly, contributes to overregulation, may inhibit
Answer (A) is incorrect because the Wagner Act of innovation, increases inflation and may place a
1935 prohibited employers from interfering with the disproportionate burden on small companies, thereby
right of employees to form unions. having an anticompetitive effect. Another criticism is
that regulators are perceived to have little concern for
Answer (B) is incorrect because the Taft-Hartley Act the relation of marginal benefits and costs. For
required unions to bargain in good faith. example, achieving total compliance with the
standards for treated wastewater may be twice as
Answer (C) is incorrect because the costly as attaining 95% of those standards. In other
Norris-LaGuardia Act of 1932 outlawed yellow-dog words, the relatively small improvements sometimes
contracts. sought by regulators may be viewed as excessively
costly by those who are regulated.
Answer (D) is correct. The Taft-Hartley Act placed
restraints on unions that resulted in a more even
balance of power between labor and management. [80] Source: CMA 1295 1-20
One provision prohibits unions from coercing
employees to become union members. Thus, the Answer (A) is incorrect because some regulated
closed shop was outlawed. The act also requires industries are subject to antitrust laws.
unions to bargain in good faith and provides for
mediation of disputes. The President of the USA was Answer (B) is correct. With respect to antitrust laws,
given authority to obtain an injunction against strikes regulated industries are covered unless specifically
or lockouts for an 80-day cooling-off period if the exempted by statute or the courts. For example,
national health, welfare, or security is threatened. regulated public utilities are specifically exempted
Also, secondary strikes and featherbedding are because they are viewed as natural monopolies.
prohibited.
Answer (C) is incorrect because the Justice
Department must look to statute or the courts to
[78] Source: CMA 1294 1-13 enforce antitrust laws.

Answer (A) is incorrect because the FTC was Answer (D) is incorrect because some regulated
created in 1914 by the Federal Trade Commission industries, such as utilities, are exempt from antitrust
Act. laws.

Answer (B) is incorrect because penalties were


stated in the 1890 Act. [81] Source: CMA 1295 1-21

Answer (C) is incorrect because the Clayton Act of Answer (A) is correct. The purpose of social
1914 prohibits mergers that substantially lessen regulation is to require behavior that is thought to be
competition. beneficial for other than purely economic
considerations. However, social regulation has costs,
Answer (D) is correct. The Sherman Act makes which lead to increased prices and inflation, reduced
illegal every contract, combination, or conspiracy in innovation and consumer choice, and decreased
restraint of trade in interstate or foreign commerce. productivity. Social regulation is criticized because
Some types of arrangements between competitors the marginal benefits may not equal the marginal
are found unreasonable without inquiry. These are costs.
called per se violations and include price fixing,
division of markets, group boycotts, and resale price Answer (B) is incorrect because enforcement policies
maintenance. are rarely viewed as lenient by those to whom social
legislation is directed.

[79] Source: CMA 1294 1-14 Answer (C) is incorrect because social legislation
may show a concern for the quality of life by
regulating the quality of products. tend to create a monopoly, (2) sales that prevent the
buyer from dealing with the seller's competitors, (3)
Answer (D) is incorrect because social regulation tie-in sales (requiring a buyer to take other products
often involves noncompliance penalties. in order to buy the first product), (4) price
discrimination, and (5) interlocking directorates
(directors serve on the boards of competing firms).
[82] Source: CMA 1295 1-22 The Clayton Act does not address unfair and
deceptive business practices such as false advertising.
Answer (A) is incorrect because the act does not
apply to export sales.
[85] Source: CMA 1295 1-25
Answer (B) is incorrect because cost differences
related to delivery are a justification for charging Answer (A) is correct. The acquisition of a shoe
different prices. retailer by a shoe manufacturer is an example of
vertical integration. Vertical integration is typified by a
Answer (C) is incorrect because the act applies only merger or acquisition involving companies that are in
to sales in interstate commerce. the same industry but at different levels in the supply
chain. In other words, one of the companies supplies
Answer (D) is correct. The Robinson-Patman Act of inputs for the other.
1936, an amendment to the Clayton Act, outlaws
price discrimination that would lead to restraint of Answer (B) is incorrect because a conglomerate is a
trade. Both buyer and seller can be found guilty of company made up of subsidiaries in unrelated
price discrimination under the provisions of the industries.
Robinson-Patman Act. Price differentiation between
customers is allowed if there is a difference in costs. Answer (C) is incorrect because market extension
For instance, quantity discounts are permitted if it can involves expanding into new market areas.
be shown that larger quantities can be shipped with a
cost savings. Charging competing wholesalers Answer (D) is incorrect because product extension
different prices for similar goods would be a violation involves selling additional products in the same
of the act. markets.

[83] Source: CMA 1295 1-23 [86] Source: CMA 1295 1-26

Answer (A) is incorrect because quantity discounts Answer (A) is incorrect because blue sky laws are
are not prohibited by the Sherman Act. state laws.

Answer (B) is incorrect because the Sherman Act Answer (B) is incorrect because blue sky laws are
does not apply to labor unions. state laws.

Answer (C) is incorrect because the Sherman Act Answer (C) is correct. Blue sky laws are state laws
does not prohibit mergers; only those that could lead designed to prevent fraudulent or misleading security
to restraint of trade are outlawed. issues. The name came from the fact that some of the
earliest laws prohibited "everything under the blue
Answer (D) is correct. The Sherman Act of 1890 skies which is fraudulent."
makes illegal every contract, combination, or
conspiracy in restraint of trade in interstate or foreign Answer (D) is incorrect because blue sky laws do
commerce. Some types of arrangements, called per not regulate the environment; they regulate sales of
se violations, are considered unreasonable without investment securities.
inquiry. These violations include price fixing, division
of markets, group boycotts, and resale price
maintenance. Agreeing to submit identical bids on a [87] Source: CMA 1295 1-27
government contract would be a form of price fixing,
and thus a per se violation. Answer (A) is incorrect because the Taft-Hartley Act
did not address internal affairs such as financial
records and reports.
[84] Source: CMA 1295 1-24
Answer (B) is incorrect because the Wagner Act of
Answer (A) is incorrect because this action is 1935 gave more power to unions.
prohibited by the Clayton Act.
Answer (C) is incorrect because the Securities
Answer (B) is incorrect because this action is Exchange Act of 1934 did not address reports by
prohibited by the Clayton Act. labor unions.

Answer (C) is incorrect because this circumstance is Answer (D) is correct. The Landrum-Griffin Act of
prohibited by the Clayton Act. 1959 (the Labor Management Reporting and
Disclosure Act) requires unions to maintain financial
Answer (D) is correct. The Clayton Act of 1914 records and submit reports to the federal
prohibits (1) mergers that may lessen competition or government. The intent of the act was to extend the
provisions of the National Labor Relations Act to the
internal affairs of unions to make the organizations Answer (A) is correct. Although unions have
more democratic and give members more rights. probably been as aggressive as ever in recent years,
union membership has declined. This decline is
attributable to the fact that growth has occurred in the
[88] Source: CMA 1295 1-28 service industries rather than in the manufacturing
industries that have been the traditional strongholds of
Answer (A) is incorrect because labor unions are unions.
specifically exempted from antitrust regulation.
Answer (B) is incorrect because this is not a
Answer (B) is incorrect because intrastate commerce compelling reason for the decline in union
is specifically exempted from antitrust regulation. membership.

Answer (C) is incorrect because patents and Answer (C) is incorrect because this is not a
copyrights are specifically exempted from antitrust compelling reason for the decline in union
regulation. membership.

Answer (D) is correct. Several types of entities and Answer (D) is incorrect because this is not a
contracts are exempt from antitrust regulation. These compelling reason for the decline in union
include firms not operating in interstate commerce, membership.
labor unions, regulated public utilities, patents and
copyrights, agricultural and fishing organizations,
financial institutions, transport industries, professional [92] Source: CMA 0685 1-35
baseball, and companies qualifying under the Export
Trading Company Act. The telecommunications Answer (A) is incorrect because more members of
industry is not exempt. minorities have joined unions as the barriers of
discrimination have been removed and as their overall
representation in the labor force has increased.
[89] Source: CMA 1295 1-29
Answer (B) is incorrect because women have joined
Answer (A) is incorrect because they are examples unions as the barriers of discrimination have been
of federal agencies enforcing social regulation. removed and as their overall representation in the
labor force has increased.
Answer (B) is incorrect because they are examples of
federal agencies enforcing social regulation. Answer (C) is correct. Because the growth areas of
the economy have been in the service industries, there
Answer (C) is incorrect because they are examples has been a decline in union membership among
of federal agencies enforcing social regulation. blue-collar workers, who are largely in manufacturing
and maintenance occupations.
Answer (D) is correct. Social regulation is intended
to benefit society as a whole. The Food and Drug Answer (D) is incorrect because public sector
Administration (FDA), National Highway Traffic employment continues to grow while manufacturing
Safety Administration, Consumer Product Safety sector employment declines.
Commission, Occupational Safety and Health
Administration (OSHA), Environmental Protection
Agency (EPA), and Equal Employment Opportunity [93] Source: Publisher
Commission (EEOC) are all examples of social
agencies. Answer (A) is incorrect because a perfectly
competitive market was envisioned by classical
economics. Modern monopolies and oligopolies often
[90] Source: CMA 1295 1-30 must be subject to regulation to enforce their social
responsibilities since the limits imposed by the market
Answer (A) is incorrect because the FTC is not are ineffective.
concerned with the regulation of foreign trade.

Answer (B) is incorrect because the FTC is not Answer (B) is incorrect because the concept
concerned with the regulation of foreign trade. embraces the public or societal interest.

Answer (C) is correct. The FTC Act of 1914 Answer (C) is correct. The concept of corporate
prohibits unfair methods of competition and unfair or social responsibility involves more than serving the
deceptive acts in commerce. The basic objectives are interests of the organization and its shareholders.
to initiate antitrust actions and protect the consumer Rather, it is an extension of responsibility to embrace
public. service to the public interest in such matters as
environmental protection, employee safety, civil
Answer (D) is incorrect because price discrimination rights, and community involvement.
and unfair trade practices are merely elements of the
overall mission of the FTC. Answer (D) is incorrect because the concept
embraces the public or societal interest.

[91] Source: CMA 0685 1-34


[94] Source: Publisher

Answer (A) is incorrect because such behavior may [97] Source: CMA 0696 1-22
prevent governmental action.
Answer (A) is incorrect because the closed shop was
Answer (B) is incorrect because it is an argument for prohibited by the Taft-Hartley Act of 1947.
such behavior.
Answer (B) is correct. The Clayton Act of 1914
Answer (C) is incorrect because it is an argument for prohibits price discrimination if it directly or indirectly
such behavior. lessens competition such that it tends to create a
monopoly. The Robinson-Patman Act amended the
Answer (D) is correct. Socially responsible behavior Clayton Act to prohibit sellers of goods from
clearly has immediate costs to the entity, for example, granting, and buyers from inducing, unfair discounts
the expenses incurred in affirmative action programs, and other preferences. However, price differentials
pollution control, and improvements in worker safety. are justified by a cost savings to the seller or a good
When one firm incurs such costs and its competitor faith effort to meet, but not undercut, the lawful price
does not, the other may be able to sell its products or of a competitor.
services more cheaply and increase its market share
at the expense of the socially responsible firm. The Answer (C) is incorrect because group boycotts
rebuttal argument is that in the long run the socially were prohibited by the Sherman Act of 1890.
responsible company may maximize profits by
creating goodwill and avoiding or anticipating Answer (D) is incorrect because oligopolies are not
governmental regulation. prohibited by the Clayton Act.

[95] Source: CMA 0696 1-18 [98] Source: CMA 0696 1-23

Answer (A) is incorrect because horizontal price Answer (A) is incorrect because interlocking
fixing occurs between two competitors at the same directorates are prohibited by the Clayton Act.
level in the distribution process (e.g., between two
retailers in the same industry). Answer (B) is incorrect because price discrimination
is prohibited by the Clayton Act as amended by the
Answer (B) is correct. Resale price maintenance Robinson-Patman Act.
agreements are a form of vertical price fixing because
a manufacturer or wholesaler restricts the price that Answer (C) is incorrect because tying contracts are
may be charged by a retailer. Such agreements are prohibited by the Clayton Act.
per se violations of the Sherman Act of 1890.
Answer (D) is correct. The Federal Trade
Answer (C) is incorrect because preemptive buying is Commission Act of 1914, as amended by the
not associated with resale price maintenance. Wheeler-Lea Amendment of 1938, prohibits unfair
methods of competition and unfair or deceptive acts
Answer (D) is incorrect because tying arrangements in commerce, including false advertising. The act also
require a buyer to purchase other products in created the Federal Trade Commission (FTC) to
addition to the one that is desired. enforce the requirements of the act. The basic
objectives of the FTC are to initiate antitrust actions
and to protect the consumer public.

[96] Source: CMA 0696 1-21


[99] Source: CMA 0696 1-24
Answer (A) is incorrect because the SEC does not
have to approve a trust indenture. Answer (A) is correct. The CPSC publishes product
safety standards, issues rules banning certain
Answer (B) is incorrect because the 1933 act hazardous products, brings federal suits to eliminate
requires disclosure of nonexempted new issuances of dangers presented by imminently hazardous
securities, including those of public utility holding consumer products, and requires manufacturers,
companies, not registration of particular entities. distributors, and retailers to give notice if they have
reason to know that their products present a
Answer (C) is correct. The Securities Act of 1933 substantial hazard. Remedies include injunctive relief,
was designed to provide complete and fair disclosure seizure of products, and civil and criminal penalties.
to potential investors. The 1933 act applies only to
the initial issuance of securities. Disclosure is Answer (B) is incorrect because the CPSC must file
accomplished through the requirement that a suit in federal court to have a product banned or
registration statement be filed with the SEC. Once seized; it cannot ban or seize a product without a
potential investors have complete disclosure, the court order.
assumption is that they can make a reasonable
decision. Answer (C) is incorrect because the CPSC must file
suit in federal court to have a product banned or
Answer (D) is incorrect because the Securities seized; it cannot ban or seize a product without a
Exchange Act of 1934 requires registration of court order.
brokers.
Answer (D) is incorrect because the CPSC may go
beyond asking for voluntary action. [103] Source: CMA 0696 1-28

Answer (A) is incorrect because constitutional law is


[100] Source: CMA 0696 1-25
the fundamental law of a jurisdiction.
Answer (A) is incorrect because age may not be
asked under the provisions of the Age Discrimination Answer (B) is incorrect because statutory law is a
in Employment Act. body of detailed enactments by the legislative branch.

Answer (B) is incorrect because questions about a Answer (C) is correct. Administrative law is
disability may not be asked under the provisions of promulgated by the executive branch under a general
the Americans with Disabilities Act of 1990. grant of authority to an agency to regulate an industry.
Administrative law may also be promulgated under a
Answer (C) is incorrect because marital status is not specific grant of authority to an agency to make
a suitable subject for questioning under the civil rights detailed rules to achieve the objectives of a statute.
laws. For example, the IRS makes rules to carry out
specific statutes but is not given the general authority
Answer (D) is correct. The Equal Employment to make rules for the collection of revenue.
Opportunity Commission (EEOC) specifies several Administrative law may not go beyond the scope of
questions that cannot be asked of applicants during the statutes under which it is promulgated.
an employment interview. These include questions
regarding age, disability, arrest record, and marital Answer (D) is incorrect because judicial (common)
status. It is permissible to ask an applicant for law is created by the courts through the adjudication
references. of cases and the publication of the resulting opinions.

[101] Source: CMA 0696 1-26 [104] Source: CMA 1296 1-20

Answer (A) is incorrect because a closed shop Answer (A) is incorrect because the
requires an employee to be a member of the union in
order to get a job. Robinson-Patman Act concerns price discrimination,
not exclusive-dealing contracts.
Answer (B) is incorrect because a union shop
requires an employee to join the union after Answer (B) is incorrect because exclusive-dealing
employment. contracts are the subject of federal antitrust law. They
are not illegal under common law.
Answer (C) is correct. Some states have
right-to-work laws that allow employees to work at Answer (C) is incorrect because there is no
any job without union membership. Thus, employees requirement to seek approval from the FTC.
can decide not to be involved with a union even
though the union represents the workers in the shop. Answer (D) is correct. The Clayton Act of 1914
prohibits exclusive-dealing requirements. However, a
Answer (D) is incorrect because paying dues is franchise relationship is contractual, and the
equivalent to the requirements of a union shop. agreement is in force for a specified period. Because
the franchisee and the franchisor have a common
public identity, an exclusive-dealing contract is
[102] Source: CMA 0696 1-27 allowable if it is necessary to assure product quality.
Without the exclusive-dealing requirement, a
Answer (A) is incorrect because no labeling low-quality product sold by the franchisee could
requirements exist in an unregulated market. reflect badly on the reputation of the franchisor.

Answer (B) is correct. Mandatory labeling follows


the disclosure approach. The basis of this approach is [105] Source: CMA 1296 1-21
the belief that regulation is sufficient if consumers have
the information needed to make rational decisions. Answer (A) is correct. The basic purpose of the
The assumptions are that consumers are sophisticated federal securities laws in the United States, primarily
enough to use the information provided and that the Securities Act of 1933 and the Securities
sellers will have an incentive to improve their Exchange Act of 1934, is to provide complete and
products. fair disclosure to potential investors. The emphasis is
on disclosure that allows informed investors to make
Answer (C) is incorrect because stricter liability rules intelligent decisions.
promote product quality and safety, whereas labeling
requirements are primarily informational. Answer (B) is incorrect because the SEC does not
evaluate the merits of investments or define an
Answer (D) is incorrect because labeling investment-grade security.
requirements do not create marketable property
rights. An example of such a regulatory practice is the Answer (C) is incorrect because a corporation's
external trading of pollution rights. by-laws provide for voting rights. For example,
preferred stock rarely has a vote.
specific laws. Otherwise, the legislation or agency
Answer (D) is incorrect because the federal laws terminates at the end of a specified period of time.
apply in all states regardless of the existence of For example, the first federal Cost Accounting
applicable state laws. Standards Board was established with a 10-year life.
At the end of that period, the agency was terminated
because Congress felt that its purpose had been
[106] Source: CMA 1296 1-22 accomplished.

Answer (A) is incorrect because the FTC was


established by the Federal Trade Commission Act of [109] Source: CMA 1296 1-25
1914.
Answer (A) is incorrect because shareholders in
Answer (B) is correct. The Sherman Act of 1890 publicly traded U. S. corporations have the right to
makes illegal every contract, combination, or vote on fundamental corporate changes, to receive
conspiracy in restraint of trade in interstate or foreign declared dividends and annual reports, to vote, to
commerce. Some types of arrangements between exercise any preemptive right that may have been
competitors are to be considered unreasonable granted, to attend meetings, to inspect corporate
without inquiry. These are known as per se violations. records, and to bring shareholder suits.
Price fixing, division of markets, group boycotts, and
resale price maintenance are per se violations. Answer (B) is incorrect because shareholders in
publicly traded U. S. corporations have the right to
Answer (C) is incorrect because price discrimination vote on fundamental corporate changes, to receive
is prohibited by the Robinson-Patman Act of 1936, declared dividends and annual reports, to vote, to
which was an amendment to the Clayton Act of exercise any preemptive right that may have been
1914. granted, to attend meetings, to inspect corporate
records, and to bring shareholder suits.
Answer (D) is incorrect because patent laws were in
effect long before the Sherman Act of 1890. Thus, Answer (C) is incorrect because shareholders in
the U.S. Constitution (Article I, Section 8) grants to publicly traded U. S. corporations have the right to
Congress the power "To promote the progress of vote on fundamental corporate changes, to receive
science and useful arts, by securing for limited times declared dividends and annual reports, to vote, to
to authors and inventors the exclusive right to their exercise any preemptive right that may have been
respective writings and discoveries." granted, to attend meetings, to inspect corporate
records, and to bring shareholder suits.

[107] Source: CMA 1296 1-23 Answer (D) is correct. A corporation is owned by
shareholders who elect a board of directors to
Answer (A) is incorrect because the Clayton Act of manage the company. The board of directors then
1914 prohibits interlocking directorates in competing hires managers to supervise operations. Shareholders
companies. do not vote on major management changes because
the powers of the board include selection and
Answer (B) is incorrect because the removal of officers and the setting of management
Robinson-Patman Act of 1936 prohibits price compensation. Shareholders do have the right to vote
discrimination. on fundamental corporate changes, e.g., mergers and
acquisitions, any changes in the corporate charter and
Answer (C) is correct. The Antitrust Improvements bylaws, and dissolution.
Act of 1976 requires corporations with annual sales
or assets exceeding $100,000,000 to give advance
notice to the Justice Department and the FTC of any [110] Source: CMA 1296 1-29
acquisition of a corporation with annual sales or
assets of $10,000,000 or more. Answer (A) is incorrect because the ADA prohibits
employers from inquiring about a job applicant's prior
Answer (D) is incorrect because the Sherman Act of health insurance claims.
1890 prohibits restraint of trade and monopoly.
Answer (B) is incorrect because the ADA applies to
both physical and mental impairments.
[108] Source: CMA 1296 1-24
Answer (C) is correct. The ADA requires
Answer (A) is incorrect because the Age organizations with 25 or more employees to provide
Discrimination in Employment Act provides reasonable accommodation for employees and job
protection for older workers. applicants with disabilities. The ADA bans
employment discrimination against people with mental
Answer (B) is incorrect because sunset laws are not or physical disabilities, provides tax incentives for
concerned with securities regulation. compliance costs, and requires remodeling of facilities
to provide access by individuals with disabilities.
Answer (C) is incorrect because laws that prohibit
the sale of alcohol on Sundays are blue laws. Another provision of the ADA is that employers are
prohibited from inquiring into a job applicant's
Answer (D) is correct. Sunset laws are provisions disability with questions concerning medical history,
that require periodic review and reenactment of prior workers' compensation or health insurance
claims, work absenteeism due to illness, past Act of 1950 prohibits the acquisition of the stock or
treatment for alcoholism, or mental illness. assets of another business if the effect may be to
lessen competition substantially or to create a
Answer (D) is incorrect because the ADA provides monopoly.
no federal funds for implementation of its provisions.

Answer (B) is incorrect because the Federal Trade


[111] Source: CMA 1296 1-30 Commission Act of 1914 prohibits unfair methods of
competition in or affecting interstate commerce. It
Answer (A) is incorrect because monetary penalties also created the Federal Trade Commission, which
were imposed for insider trading before 1988. For has authority to enforce the Clayton and
example, the Securities Exchange Act of 1934 Robinson-Patman Acts. The FTC also is authorized
requires insiders to turn over to the corporation any to proceed against unfair or deceptive acts or
short-swing profits on purchases and sales of practices.
corporate stock.
Answer (C) is incorrect because the Sherman Act of
1890 makes illegal every contract, combination, or
Answer (B) is correct. The Insider Trading Sanctions conspiracy that unreasonably restrains trade in
Act of 1984 extended the law to include those who interstate or foreign commerce. It also prohibits
aid and abet insider trading. It also imposed a monopolization and attempts and conspiracies to
treble-damages penalty. The 1988 act increased monopolize.
prison terms from 5 to 10 years, the maximum fine
for individuals to $1,000,000, and the maximum fine Answer (D) is correct. The Robinson-Patman Act of
for corporations to $2,500,000. In addition, the SEC 1936 amended the Clayton Act with respect to price
now has authority to reward an informant with up to discrimination. Price discrimination by both buyers
10% of the fine levied against the perpetrator. and sellers is prohibited in interstate commerce of
goods of like grade and quality. The purpose of the
Answer (C) is incorrect because, under section 10(b) act is to protect competition. However, price
of the 1934 act and Rule 10b-5, insider trading is a differentials are allowed if justified by a cost savings
fraudulent act subject to criminal penalties. to the seller or a good-faith effort to meet a
competitor's lawful price.
Answer (D) is incorrect because the SEC already
held the right to recover gains resulting from the illegal
use of insider information. [114] Source: CMA 0697 1-26

Answer (A) is incorrect because the Federal Trade


[112] Source: CMA 0697 1-22 Commission Act of 1914 prohibits unfair methods of
competition in or affecting interstate commerce. It
Answer (A) is correct. SEC Rule 415 allows also created the Federal Trade Commission, which
corporations to file registration statements covering a has authority to enforce the Clayton and
stipulated amount of securities that may be issued Robinson-Patman Acts. The FTC also is authorized
over the 2-year effective period of the statement. The to proceed against unfair or deceptive acts or
securities are placed on the shelf and issued at an practices.
opportune moment without the necessity of filing a
new registration statement, observing a 20-day Answer (B) is correct. The Securities Exchange Act
waiting period, or preparing a new prospectus. The of 1934 addresses the issue of insider trading.
issuer is only required to provide updating Specifically, insiders must turn over to the
amendments or to refer investors to quarterly and corporation any profits earned on purchases and
annual statements filed with the SEC. Shelf sales of their company's stock that fall within six
registration is most advantageous to large months of each other. They are also prohibited from
corporations that frequently offer securities to the buying or selling stock based on inside information
public. not available to the public.

Answer (B) is incorrect because an indenture is a Answer (C) is incorrect because the Clayton Act of
contract associated with the issuance of a debt 1914 was intended to prevent monopolies. A
instrument. probability of a significant anticompetitive effect is a
basis for most violations of the act. It specifically
Answer (C) is incorrect because a secondary market addresses price discrimination, tying contracts,
registration does not expedite the issuance of exclusive dealing arrangements, mergers, and
securities. interlocking directorates.

Answer (D) is incorrect because a red-herring is a Answer (D) is incorrect because the North American
preliminary prospectus issued during the 20-day Free Trade Agreement is an agreement providing for
waiting period before the related registration free trade among the USA, Canada, and Mexico.
statement becomes effective.

[115] Source: CMA 0697 1-27


[113] Source: CMA 0697 1-25
Answer (A) is incorrect because on-time departure
Answer (A) is incorrect because the Celler-Kefauver standards are an aspect of the airline industry that has
been deregulated. is essentially a disclosure statute. The SEC does not
evaluate the merits of securities. Its role is to enforce
Answer (B) is incorrect because domestic fare the laws ensuring the public availability of information
schedules are an aspect of the airline industry that has to potential investors.
been deregulated.

Answer (C) is correct. Airline operations have been [118] Source: Publisher
almost fully deregulated by the U. S. government, for
example, with regard to departure standards, fare Answer (A) is incorrect because the definition of the
schedules, and the opportunity to eliminate term "hazardous substance" as used in CERCLA
does not include petroleum or any derivatives thereof
unprofitable routes. Despite deregulation, however, or natural gas.
airlines are still subject to federal and state labor
laws, including those governing collective bargaining. Answer (B) is incorrect because the definition of the
The National Labor Relations Board exercises its term "hazardous substance" as used in CERCLA
jurisdiction over all transportation enterprises that does not include petroleum or any derivatives thereof
furnish interstate services. or natural gas.

Answer (D) is incorrect because international fare Answer (C) is correct. Asbestos is a hazardous
schedules are an aspect of the airline industry that has substance subject to the provisions of CERCLA.
been deregulated.
Answer (D) is incorrect because the definition of the
term "hazardous substance" as used in CERCLA
[116] Source: CMA 0697 1-28 does not include petroleum or any derivatives thereof
or natural gas.
Answer (A) is correct. Federal administrative
agencies belong to the executive branch of
government. They have quasi-legislative powers [119] Source: Publisher
(rulemaking authority pursuant to enabling statutes
enacted by Congress), quasi-judicial powers (the Answer (A) is incorrect because under NEPA,
ability to adjudicate certain disputes within their federal agencies must give environmental
jurisdiction), and quasi-executive powers (for considerations a weight equal to, but not greater than,
example, to investigate and prosecute violations). that afforded nonenvironmental concerns.
However, no federal agency has taxing authority, a
power reserved to Congress. Moreover, all bills for Answer (B) is correct. The provisions of NEPA
raising revenue must originate in the House of focus on federal governmental actions. Federal
Representatives. agencies are specifically directed to incorporate an
analysis of environmental consequences in their
Answer (B) is incorrect because it is a frequently decision-making processes. Actions of private
heard criticism of federal regulatory agencies and persons are affected by NEPA only when federal
policies. involvement (approval, funding, etc.) is necessary
before such persons may act (e.g., federal approval
Answer (C) is incorrect because it is a frequently before drilling for oil in ocean waters within U.S.
heard criticism of federal regulatory agencies and jurisdiction). Otherwise, NEPA does not directly
policies. concern activities of private persons.

Answer (D) is incorrect because it is a frequently Answer (C) is incorrect because under NEPA,
heard criticism of federal regulatory agencies and federal agencies must give environmental
policies. considerations a weight equal to, but not greater than,
that afforded nonenvironmental concerns.

[117] Source: CMA 0697 1-29 Answer (D) is incorrect because NEPA augments the
existing powers of federal agencies to deal with these
Answer (A) is incorrect because insider trading is environmental matters.
prohibited by Section 10(b) of the Securities
Exchange Act of 1934 and by the SEC's Rule 10b-5.
[120] Source: Publisher
Answer (B) is incorrect because an issuer that wishes
to make an interstate offering of new securities to the Answer (A) is incorrect because the CWA broadly
public must file a registration statement with the SEC. prohibits any discharges of pollutants into waters,
except if in compliance with the act. Impairment of
Answer (C) is incorrect because the Securities navigation is irrelevant.
Exchange Act of 1934 provides rules with regard to
proxy solicitations and tender offers. Answer (B) is incorrect because, to be subject to the
CWA, the waters must be "navigable waters."
Answer (D) is correct. The SEC is charged with
enforcement of federal securities laws. Under the Answer (C) is incorrect because the Rivers and
securities Act of 1933, the offer or sale of a security Harbors Act of the late 1800s was used to combat
to the public requires registration with the SEC pollutive discharges, although its original purpose was
absent a specific exemption. However, the 1933 act to keep waterways clear from obstructions to
navigation. rights, and community involvement.

Answer (D) is correct. The CWA (1972) Answer (D) is incorrect because the concept
substantially amended the Federal Water Pollution embraces the public or societal interest.
Control Act of 1948. It seeks to restore and maintain
the physical and biological integrity of the waters of
the United States. Its objectives are to render water [124] Source: Publisher
suitable for recreation and propagation of fish and
other wildlife and to eliminate discharges of Answer (A) is incorrect because such behavior may
pollutants. prevent governmental action.

Answer (B) is incorrect because each is an argument


[121] Source: CMA 1294 1-9 for such behavior.

Answer (A) is incorrect because no information is Answer (C) is incorrect because each is an argument
given regarding costs. Only selling prices and for such behavior.
quantities are given.
Answer (D) is correct. Socially responsible behavior
Answer (B) is incorrect because AC represents clearly has immediate costs to the entity, for example,
greater satisfied demand. the expenses incurred in affirmative action programs,
pollution control, and improvements in worker safety.
Answer (C) is correct. The area ABC represents the When one firm incurs such costs and its competitor
efficiency gain from antitrust action. It is the increase does not, the other may be able to sell its products or
in productivity (benefits to society) from the increased services more cheaply and increase its market share
quantity and lower prices resulting from the antitrust at the expense of the socially responsible firm. The
action. rebuttal argument is that in the long run the socially
responsible company may maximize profits by
Answer (D) is incorrect because total benefit is creating goodwill and avoiding or anticipating
represented by the greater quantity produced (area governmental regulation.
CBQ2Q1).

[125] Source: Publisher


[122] Source: Publisher
Answer (A) is incorrect because it states an aspect of
Answer (A) is incorrect because "practitioners of the competence requirement.
management accounting and financial management
have an obligation to the public, their profession, the Answer (B) is correct. According to the IMA Code
organization they serve, and themselves, to maintain of Ethics, financial managers/management
the highest standards of ethical conduct." accountants must "avoid actual or apparent conflicts
of interest and advise all appropriate parties of any
Answer (B) is incorrect because the audit committee potential conflict."
would be consulted first only if it were the next higher
managerial level. Answer (C) is incorrect because it states an aspect of
the confidentiality requirement.
Answer (C) is correct. To resolve an ethical problem,
the financial manager/management accountant's first Answer (D) is incorrect because it states an aspect of
step is usually to consult his/her immediate superior. If the competence requirement.
that individual is involved, the matter should be taken
to the next higher level of management.
[126] Source: Publisher
Answer (D) is incorrect because if the superior is
involved, the next higher managerial level should be Answer (A) is incorrect because the code does not
consulted first. address these matters.

Answer (B) is incorrect because the code does not


[123] Source: Publisher address these matters.

Answer (A) is incorrect because a perfectly Answer (C) is correct. Financial


competitive market was envisioned by classical managers/management accountants may not dis close
economics. confidential information acquired in the course of their
work unless authorized or legally obligated to do so.
Answer (B) is incorrect because the concept They must inform subordinates about the
embraces the public or societal interest. confidentiality of information and monitor their
activities to maintain that confidentiality. Moreover,
Answer (C) is correct. The concept of corporate financial managers/management accountants should
social responsibility involves more than serving the avoid even the appearance of using confidential
interests of the organization and its shareholders. information to their unethical or illegal advantage.
Rather, it is an extension of responsibility to embrace
service to the public interest in such matters as Answer (D) is incorrect because other employment
environmental protection, employee safety, civil may be accepted unless it constitutes a conflict of
interest.

[127] Source: CMA 1

Answer (A) is incorrect because the competence


standard pertains to the financial
manager/management accountant's responsibility to
maintain his/her professional skills and knowledge. It
also pertains to the performance of activities in a
professional manner.

Answer (B) is incorrect because the confidentiality


standard concerns the financial manager/management
accountant's responsibility not to disclose or use the
firm's confidential information.

Answer (C) is correct. One of the responsibilities of


the financial manager/management accountant under
the integrity standard is to "recognize and
communicate professional limitations or other
constraints that would preclude responsible judgment
or successful performance of an activity."

Answer (D) is incorrect because objectivity is the


fourth part of the IMA Code of Ethics. It requires
that information be communicated "fairly and
objectively," and that all information that could
reasonably influence users be fully disclosed.

[128] Source: CMA 2

Answer (A) is incorrect because the competence


standard pertains to the financial
manager/management accountant's responsibility to
maintain his/her professional skills and knowledge. It
also pertains to the performance of activities in a
professional manner.

Answer (B) is incorrect because the confidentiality


standard concerns the financial manager/management
accountant's responsibility not to disclose or use the
firm's confidential information.

Answer (C) is correct. The integrity standard requires


the financial manager/management accountant to
"refuse any gift, favor, or hospitality that would
influence or would appear to influence his/her actions.

Answer (D) is incorrect because objectivity is the


fourth part of the IMA Code of Ethics. It requires
that information be communicated "fairly and
objectively," and that all information that could
reasonably influence users be fully disclosed.

You might also like